Modules 1-3: Assessment Techniques & General Survey, Eyes, Ears, Nose, Mouth, & Throat, Lungs, Chest wall, Thorax, Heart & Neck Vessels Peripheral Vascular and Lymphatics Head, & Neck ,

अब Quizwiz के साथ अपने होमवर्क और परीक्षाओं को एस करें!

A patient has been admitted with chronic arterial symptoms. During the assessment, the nurse should expect which findings? Select all that apply. a. The patient has a history of diabetes and cigarette smoking. b. The patient's skin is pale and cool. c. The patient's ankles have two small, weeping ulcers. d. The patient works long hours sitting at a computer desk. e. The patient states that the pain gets worse when walking. f. The patient states that the pain is worse at the end of the day.

A, B, E See Table 20-3. Patients with chronic arterial symptoms often have a history of smoking and diabetes (among other risk factors). The pain has a gradual onset, with exertion, and is relieved with rest or dangling. The skin appears cool and pale. The other responses reflect chronic venous problems

The nurse is assessing voice sounds during a respiratory assessment. Which of these findings indicates a normal assessment? Select all that apply. a. Voice sounds are faint, muffled, and almost inaudible when the patient whispers "one, two, three" in a very soft voice. b. As the patient says "ninety-nine" repeatedly, the examiner hears the words "ninety-nine" clearly. c. When the patient speaks in a normal voice, the examiner can hear a sound but cannot distinguish exactly what is being said. d. As the patient says a long "ee-ee-ee" sound, the examiner also hears a long "ee-ee-ee" sound. e. As the patient says a long "ee-ee-ee" sound, the examiner hears a long "aaaaaa" sound.

A, C, D As a patient says "ninety-nine" repeatedly, normally, the examiner hears sound but cannot distinguish what is being said. If a clear "ninety-nine" is auscultated, then it could indicate increased lung density, which enhances transmission of voice sounds. This is a measure of bronchophony. When a patient says a long "ee-ee-ee" sound, normally the examiner also hears a long "ee-ee-ee" sound through auscultation. This is a measure of egophony. If the examiner hears a long "aaaaaa" sound instead, this could indicate areas of consolidation or compression. With whispered pectoriloquy, as when a patient whispers a phrase such as "one-two-three," the normal response when auscultating voice sounds is to hear sounds that are faint, muffled, and almost inaudible. If the examiners hears the whispered voice clearly, as if the patient is speaking through the stethoscope, then consolidation of the lung fields may exist.

A patient is recovering from several hours of orthopedic surgery. During an assessment of the patient's lower legs, the nurse will monitor for signs of acute venous symptoms. Signs of acute venous symptoms include which of the following? Select all that apply. a. Intense, sharp pain, with the deep muscle tender to touch b. Aching, tired pain, with a feeling of fullness c. Pain is worse at the end of the day d. Sudden onset e. Warm, red, and swollen calf f. Pain that is relieved with elevation of leg

A, D, E Signs and symptoms of acute venous problems include pain in the calf that has a sudden onset and that is intense and sharp with tenderness in the deep muscle when touched. The calf is warm, red, and swollen. The other options are symptoms of chronic venous problems.

The nurse is preparing to percuss the abdomen of a patient. The purpose of the percussion is to assess the underlying tissue: a. turgor. b. texture. c. density. d. consistency.

C Percussion yields a sound that depicts the location, size, and density of the underlying organ. Turgor and texture are assessed with palpation

A patient has been diagnosed with strep throat. The nurse is aware that without treatment, which complication may occur? a. Rubella b. Leukoplakia c. Rheumatic fever d. Scarlet fever

C Untreated strep throat may lead to rheumatic fever. When performing a health history, ask whether the patient's sore throat was documented as streptococcal.

When auscultating the lungs of an adult patient, the nurse notes that over the posterior lower lobes low-pitched, soft breath sounds are heard, with inspiration being longer than expiration. The nurse interprets that these are: a. sounds normally auscultated over the trachea. b. bronchial breath sounds and are normal in that location. c. vesicular breath sounds and are normal in that location. d. bronchovesicular breath sounds and are normal in that location.

C Vesicular breath sounds are low-pitched, soft sounds with inspiration being longer than expiration. These breath sounds are expected over peripheral lung fields where air flows through smaller bronchioles and alveoli.

A 10-year-old is at the clinic for "a sore throat lasting 6 days." The nurse is aware that which of these findings would be consistent with an acute infection? a. Tonsils 1+/1-4+ and pink, same color as oral mucosa b. Tonsils 2+/1-4+ with small plugs of white debris c. Tonsils 3+/1-4+ with large white spots d. Tonsils 3+/1-4+ with pale coloring

C With an acute infection, tonsils are bright red and swollen and may have exudate or large white spots. Tonsils are enlarged to 2+, 3+, or 4+ with an acute infection.

The nurse is testing a patient's visual accommodation, which refers to which action? a. Pupillary constriction when looking at a near object b. Pupillary dilation when looking at a far object c. Changes in peripheral vision in response to light d. Involuntary blinking in the presence of bright light

A The muscle fibers of the iris contract the pupil in bright light and accommodate for near vision, which also results in pupil constriction. The other responses are not correct.

The nurse is assessing a patient's pulses and notices a difference between the patient's apical pulse and radial pulse. The apical pulse was 118 beats per minute, and the radial pulse was 105 beats per minute. What is the pulse deficit?

13 The nurse should count a serial measurement (one after the other) of apical beat and then the radial pulse. Normally every beat heard at the apex should perfuse to the periphery and be palpable. The two counts should be identical. If different, the nurse should subtract the radial rate from the apical and record the remainder as the pulse deficit.

The nurse is percussing over the lungs of a patient with pneumonia. The nurse knows that percussion over an area of atelectasis in the lungs would reveal: a. dullness. b. tympany. c. resonance. d. hyperresonance.

A A dull percussion note signals an abnormal density in the lungs, as with pneumonia, pleural effusion, atelectasis, or tumor.

During a cardiovascular assessment, the nurse knows that a "thrill" is: a. a vibration that is palpable. b. palpated in the right epigastric area. c. associated with ventricular hypertrophy. d. a murmur auscultated at the third intercostal space.

A A thrill is a palpable vibration. It signifies turbulent blood flow and accompanies loud murmurs. The absence of a thrill does not rule out the presence of a murmur.

The nurse is examining a patient's retina with an ophthalmoscope. Which finding is considered normal? a. An optic disc that is a yellow-orange color b. Optic disc margins that are blurred around the edges c. The presence of pigmented crescents in the macular area d. The presence of the macula located on the nasal side of the retina

A The optic disc is located on the nasal side of the retina. It is a creamy yellow-orange to pink color, and the edges are distinct and sharply demarcated, not blurred. A pigmented crescent is black, and it is due to the accumulation of pigment in the choroid.

When examining the eye, the nurse notices that the patient's eyelid margins approximate completely. The nurse recognizes that this assessment finding: a. is expected. b. may indicate a problem with extraocular muscles. c. may result in problems with tearing. d. indicates increased intraocular pressure.

A The palpebral fissure is the elliptical open space between the eyelids, and, when closed, the lid margins approximate completely, which is a normal finding.

The sac that surrounds and protects the heart is called the: a. pericardium. b. myocardium. c. endocardium. d. pleural space.

A The pericardium is a tough fibrous double-walled sac that surrounds and protects the heart. It has two layers that contain a few milliliters of serous pericardial fluid.

The nurse suspects that a patient has hyperthyroidism and laboratory data indicate that the patient's thyroxine and tri-iodothyronine hormone levels are elevated. Which of these findings would the nurse most likely find on examination? a. Tachycardia b. Constipation c. Rapid dyspnea d. Atrophied nodular thyroid

A Thyroxine and tri-iodothyronine are thyroid hormones that stimulate the rate of cellular metabolism, resulting in tachycardia. With an enlarged thyroid as in hyperthyroidism, the nurse might expect to find diffuse enlargement (goiter) or a nodular lump, but not an atrophied gland. Dyspnea and constipation are not findings associated with hyperthyroidism.

The nurse is listening to the breath sounds of a patient with severe asthma. Air passing through narrowed bronchioles would produce which of these adventitious sounds? a. Wheezes b. Bronchial sounds c. Bronchophony d. Whispered pectoriloquy

A Wheezes are caused by air squeezed or compressed through passageways narrowed almost to closure by collapsing, swelling, secretions, or tumors, such as with acute asthma or chronic emphysema.

Which of these statements is true regarding the arterial system? a. Arteries are large-diameter vessels. b. The arterial system is a high-pressure system. c. The walls of arteries are thinner than those of veins.

B The pumping heart makes the arterial system a high-pressure system.

A patient comes to the clinic complaining of a cough that is worse at night but not as bad during the day. The nurse recognizes that this may indicate: a. pneumonia. b. postnasal drip or sinusitis. c. exposure to irritants at work. d. chronic bronchial irritation from smoking.

B A cough that occurs mainly at night may indicate postnasal drip or sinusitis. Exposure to irritants at work causes an afternoon or evening cough. Smokers experience early morning coughing. Coughing associated with acute illnesses such as pneumonia is continuous throughout the day.

Which of these veins are responsible for most of the venous return in the arm? a. Deep b. Ulnar c. Subclavian d. Superficial

D The superficial veins of the arms are in the subcutaneous tissue and are responsible for most of the venous return.

1.​The nurse is preparing to perform a physical assessment. Which statement is true about the inspection phase of the physical assessment? a. Inspection usually yields little information. b. Inspection takes time and reveals a surprising amount of information. c. Inspection may be somewhat uncomfortable for the expert practitioner. d. Inspection requires a quick glance at the patient's body systems before proceeding on with palpation.

​B A focused inspection takes time and yields a surprising amount of information. Initially, the examiner may feel uncomfortable "staring" at the person without also "doing something." A focused assessment is much more than a "quick glance."

The nurse needs to palpate the temporomandibular joint for crepitation. This joint is located just below the temporal artery and anterior to the: a. hyoid. b. vagus nerve. c. tragus. d. mandible.

C The temporomandibular joint is just below the temporal artery and anterior to the tragus.

The nurse is preparing for a class on risk factors for hypertension, and reviews recent statistics. Which racial group has the highest prevalence of hypertension in the world? a. African-Americans b. Whites c. American Indians d. Hispanics

A According to the American Heart Association, the prevalence of hypertension is higher among African-Americans than in other racial groups.

The nurse is reviewing venous blood flow patterns. Which of these statements best describes the mechanism(s) by which venous blood returns to the heart? a. Intraluminal valves ensure unidirectional flow toward the heart. b. Contracting skeletal muscles milk blood distally toward the veins. c. The high-pressure system of the heart helps to facilitate venous return. d. Increased thoracic pressure and decreased abdominal pressure facilitate venous return to the heart.

A Blood moves through the veins by (1) contracting skeletal muscles that milk the blood proximally; (2) pressure gradients caused by breathing, in which inspiration makes the thoracic pressure decrease and the abdominal pressure increase; and (3) the intraluminal valves, which ensure unidirectional flow toward the heart.

During an examination of a 3-year-old child, the nurse notices a bruit over the left temporal area. The nurse should: a. continue the examination because this is a normal finding for this age. b. check for the bruit again in 1 hour. c. notify the parents that a bruit has been detected in their child. d. stop the examination and notify the physician.

A Bruits are common in the skull in children under 4 or 5 years of age and in children with anemia. They are systolic or continuous and are heard over the temporal area.

A 2-week-old infant can fixate on an object but cannot follow a light or bright toy. The nurse would: a. consider this a normal finding. b. assess the pupillary light reflex for possible blindness. c. continue with the examination and assess visual fields. d. expect that a 2-week-old infant should be able to fixate and follow an object.

A By 2 to 4 weeks an infant can fixate on an object. By the age of 1 month, the infant should fix-ate and follow a bright light or toy.

The nurse is performing an assessment on a 7-year-old child who has symptoms of chronic watery eyes, sneezing, and clear nasal drainage. The nurse notices the presence of a transverse line across the bridge of the nose, dark blue shadows below the eyes, and a double crease on the lower eyelids. These findings are characteristic of: a. allergies. b. a sinus infection. c. nasal congestion. d. an upper respiratory infection.

A Chronic allergies often develop chronic facial characteristics. These include blue shadows below the eyes, a double or single crease on the lower eyelids, open-mouth breathing, and a transverse line on the nose.

During auscultation of the lungs, the nurse expects decreased breath sounds to be heard in which situation? a. When the bronchial tree is obstructed b. When adventitious sounds are present c. In conjunction with whispered pectoriloquy d. In conditions of consolidation, such as pneumonia

A Decreased or absent breath sounds occur when the bronchial tree is obstructed, as in emphysema, and when sound transmission is obstructed, as in pleurisy, pneumothorax, or pleural effusion.

A 25-year-old woman in her fifth month of pregnancy has a blood pressure of 100/70 mm Hg. In reviewing her previous exam, the nurse notes that her blood pressure in her second month was 124/80 mm Hg. In evaluating this change, what does the nurse know to be true? a. This is the result of peripheral vasodilatation and is an expected change. b. Because of increased cardiac output, the blood pressure should be higher this time. c. This is not an expected finding because it would mean a decreased cardiac output. d. This would mean a decrease in circulating blood volume, which is dangerous for the fetus.

A Despite the increased cardiac output, arterial blood pressure decreases in pregnancy because of peripheral vasodilatation. The blood pressure drops to its lowest point during the second trimester and then rises after that.

During an assessment of a 20-year-old patient with a 3-day history of nausea and vomiting, the nurse notices dry mucosa and deep vertical fissures in the tongue. These findings are reflective of: a. dehydration. b. irritation by gastric juices. c. a normal oral assessment. d. side effects from nausea medication.

A Dry mouth occurs with dehydration or fever. The tongue has deep vertical fissures.

During auscultation of breath sounds, the nurse should use the stethoscope correctly, in which of the following ways? a. Listen to at least one full respiration in each location. b. Listen as the patient inhales and then go to the next site during exhalation. c. Have the patient breathe in and out rapidly while the nurse listens to the breath sounds. d. If the patient is modest, listen to sounds over his or her clothing or hospital gown.

A During auscultation of breath sounds with a stethoscope, it is important to listen to one full respiration in each location. During the examination, the nurse should monitor the breathing and offer times for the person to breathe normally to prevent possible dizziness.

The nurse is reviewing percussion techniques with a newly graduated nurse. Which technique, if used by the new nurse, indicates that more review is needed? The nurse: a. percusses once over each area. b. lifts the striking finger off quickly after each stroke. c. strikes with the finger tip, not the finger pad. d. uses the wrist to make the strikes, not the arm.

A For percussion, the nurse should percuss two times over each location. The striking finger should be lifted off quickly because a resting finger damps off vibrations. The tip of the striking finger should make contact, not the pad of the finger. The wrist must be relaxed, and it is used to make the strikes, not the arm.

During a well-baby checkup, a mother is concerned because her 2-month-old infant cannot hold her head up when she is pulled to a sitting position. Which response by the nurse is appropriate? a. "Head control is usually achieved by 4 months of age." b. "You shouldn't be trying to pull your baby up like that until she is older." c. "This is a concern because head control should be achieved by this time." d. "This is a concern because it indicates possible nerve damage to the neck muscles."

A Head control is achieved by 4 months when the baby can hold the head erect and steady when pulled to a vertical position. The other responses are not appropriate.

A 70-year-old patient is being seen in the clinic for severe exacerbation of his heart failure. Which of these findings is the nurse most likely to observe in this situation? a. Shortness of breath, orthopnea, paroxysmal nocturnal dyspnea, ankle edema b. Rasping cough, thick mucoid sputum, wheezing, bronchitis c. Productive cough, dyspnea, weight loss, anorexia, tuberculosis d. Fever, dry nonproductive cough, diminished breath sounds

A Heart failure often presents with increased respiratory rate, shortness of breath on exertion, orthopnea, paroxysmal nocturnal dyspnea, nocturia, ankle edema, and pallor in light-skinned individuals. A patient with rasping cough, thick mucoid sputum, and wheezing may have bronchitis. Productive cough, dyspnea, weight loss, and dyspnea are seen with tuberculosis; fever, dry nonproductive cough, and diminished breath sounds may indicate Pneumocystis jiroveci (P. carinii) pneumonia. See Table 18-8.

How should the nurse document mild, slight pitting edema present at the ankles of a pregnant patient? a. 1+/0-4+ b. 3+/0-4+ c. 4+/0-4+ d. Brawny edema

A If pitting edema is present, then the nurse should grade it on a scale of 1+ (mild) to 4+ (severe). Brawny edema appears as nonpitting edema and feels hard to the touch.

The nurse is doing an oral assessment on a 40-year-old African-American patient and notices the presence of a 1-cm, nontender, grayish-white lesion on the left buccal mucosa. Which of these statements is true concerning this lesion? a. This lesion is leukoedema and is common in darkly pigmented persons. b. This is the result of hyperpigmentation and is normal. c. This is torus palatinus and would normally only be found in smokers. d. This type of lesion is indicative of cancer and should be tested immediately.

A Leukoedema, a grayish-white benign lesion occurring on the buccal mucosa, is seen most often in African-Americans.

A 68-year-old woman is in the eye clinic for a checkup. She tells the nurse that she has been having trouble with reading the paper, sewing, and even seeing the faces of her grandchildren. On examination, the nurse notes that she has some loss of central vision but her peripheral vision is normal. These findings suggest that: a. she may have macular degeneration. b. her vision is normal for someone her age. c. she has the beginning stages of cataract formation. d. she has increased intraocular pressure or glaucoma.

A Macular degeneration is the most common cause of blindness. It is characterized by loss of central vision. Cataracts would show lens opacity. Chronic open-angle glaucoma, the most common type of glaucoma, involves a gradual loss of peripheral vision.

When assessing tactile fremitus, the nurse recalls that it is normal to feel tactile fremitus most intensely over which location? a. Between the scapulae b. Third intercostal space, MCL c. Fifth intercostal space, MAL d. Over the lower lobes, posterior side

A Normally, fremitus is most prominent between the scapulae and around the sternum. These are sites where the major bronchi are closest to the chest wall. Fremitus normally decreases as one progress down the chest because more tissue impedes sound transmission.

The nurse is assessing a patient with a history of intravenous drug abuse. In assessing his mouth, the nurse notices a dark red confluent macule on the hard palate. This could be an early sign of: a. AIDS. b. measles. c. leukemia. d. carcinoma.

A Oral Kaposi sarcoma is a bruise-like, dark red or violet, confluent macule that usually occurs on the hard palate. It may appear on the soft palate or gingival margin. Oral lesions may be among the earliest lesions to develop with AIDS

The nurse will use which technique of assessment to determine the presence of crepitus, swelling, and pulsations? a. Palpation b. Inspection c. Percussion d. Auscultation

A Palpation applies the sense of touch to assess these factors: texture, temperature, moisture, organ location and size, as well as any swelling, vibration or pulsation, rigidity or spasticity, crepitation, presence of lumps or masses, and presence of tenderness or pain.

When percussing over the liver of a patient, the nurse notices a dull sound. The nurse should: a. consider this a normal finding. b. palpate this area for an underlying mass. c. reposition the hands and attempt to percuss in this area again. d. consider this an abnormal finding and refer the patient for additional treatment.

A Percussion over relatively dense organs, such as the liver or spleen, will produce a dull sound. The other responses are not correct.

When performing the corneal light reflex assessment, the nurse notes that the light is reflected at 2 o'clock in each eye. The nurse should: a. consider this a normal finding. b. refer the individual for further evaluation. c. document this as an asymmetric light reflex. d. perform the confrontation test to validate the findings.

A Reflection of the light on the corneas should be in exactly the same spot on each eye, or sym-metric. If asymmetry is noted, then the nurse should administer the cover test.

During an ophthalmoscopic examination of the eye, the nurse notices areas of exudate that look like "cotton wool" or fluffy gray-white cumulus clouds. This finding indicates which possible problem? a. Diabetes b. Hyperthyroidism c. Glaucoma d. Hypotension

A Soft exudates or "cotton wool" areas look like fluffy gray-white cumulus clouds, They occur with diabetes, hypertension, subacute bacterial endocarditis, lupus, and papilledema of any cause. These exudates are not found with hyperthyroidism, glaucoma, or hypotension.

The nurse is preparing to use a stethoscope for auscultation. Which statement is true regarding the diaphragm of the stethoscope? The diaphragm: a. is used to listen for high-pitched sounds. b. is used to listen for low-pitched sounds. c. should be held lightly against the person's skin to block out low-pitched sounds. d. should be held lightly against the person's skin to listen for extra heart sounds and murmurs.

A The diaphragm of the stethoscope is best for listening to high-pitched sounds such as breath, bowel, and normal heart sounds. It should be held firmly against the person's skin, firmly enough to leave a ring. The bell of the stethoscope is best for soft, low-pitched sounds such as extra heart sounds or murmurs.

The nurse is assessing the apical pulse of a 3-month-old infant and finds that the heart rate is 135 beats per minute. The nurse interprets this result as: a. normal for this age. b. lower than expected. c. higher than expected, probably as a result of crying. d. higher than expected, reflecting persistent tachycardia.

A The heart rate may range from 100 to 180 beats per minute immediately after birth and then stabilize to an average of 120 to 140 beats per minute. Infants normally have wide fluctuations with activity, from 170 beats per minute or more with crying or being active to 70 to 90 beats per minute with sleeping. Persistent tachycardia is greater than 200 beats per minute in newborns or greater than 150 beats per minute in infants.

When examining a patient's cranial nerve (CN) function, the nurse remembers that the muscles in the neck that are innervated by CN XI are the: a. sternomastoid and trapezius. b. spinal accessory and omohyoid. c. trapezius and sternomandibular. d. sternomandibular and spinal accessory.

A The major neck muscles are the sternomastoid and the trapezius. They are innervated by CN XI, the spinal accessory.

The salivary gland that is the largest and located in the cheek in front of the ear is the _____ gland. a. parotid b. Stensen's c. sublingual d. submandibular

A The mouth contains three pairs of salivary glands. The largest, the parotid gland, lies within the cheeks in front of the ear extending from the zygomatic arch down to the angle of the jaw. The Stensen's duct (not gland) drains the parotid gland onto the buccal mucosa opposite the second molar. The sublingual gland is located within the floor of the mouth under the tongue. The submandibular gland lies beneath the mandible at the angle of the jaw

A patient has hard, nonpitting edema of the left lower leg and ankle. The right leg has no edema. Based on these findings, the nurse recalls that: a. nonpitting, hard edema occurs with lymphatic obstruction. b. alterations in arterial function will cause this edema. c. phlebitis of a superficial vein will cause bilateral edema.

A Unilateral edema occurs with occlusion of a deep vein and with unilateral lymphatic obstruction. With these factors, the edema is nonpitting and feels hard to the touch (brawny edema).

The nurse is doing an assessment on a 21-year-old patient and notices that his nasal mucosa appears pale, gray, and swollen. What would be the most appropriate question to ask the patient? a. "Are you aware of having any allergies?" b. "Do you have an elevated temperature?" c. "Have you had any symptoms of a cold?" d. "Have you been having frequent nosebleeds?"

A With chronic allergy, mucosa looks swollen, boggy, pale, and gray. Elevated body temperature, colds, and nosebleeds do not cause these mucosal changes

When examining children affected with Down syndrome (trisomy 21), the nurse looks for the possible presence of: a. ear dysplasia. b. a long, thin neck. c. a protruding thin tongue. d. a narrow and raised nasal bridge.

A With the chromosomal aberration trisomy 21, also known as Down syndrome, head and face characteristics may include upslanting eyes with inner epicanthal folds, a flat nasal bridge, a small broad flat nose, a protruding thick tongue, ear dysplasia, a short broad neck with webbing, and small hands with a single palmar crease.

During percussion, the nurse knows that a dull percussion note elicited over a lung lobe most likely results from: a. shallow breathing. b. normal lung tissue. c. decreased adipose tissue. d. increased density of lung tissue.

A dull percussion note indicates an abnormal density in the lungs, as with pneumonia, pleural effusion, atelectasis, or tumor. Resonance is the expected finding in normal lung tissue.

A patient comes into the clinic complaining of pain in her right eye. On examination, the nurse sees a pustule at the lid margin that is painful to touch, red, and swollen. The nurse recognizes that this is a. a chalazion. b. a hordeolum (stye). c. dacryocystitis. d. blepharitis.

B A hordeolum, or stye, is a painful, red, and swollen pustule at the lid margin. A chalazion is a nodule protruding on the lid, toward the inside, and is nontender, firm, with discrete swelling. Dacryocystitis is an inflammation of the lacrimal sac. Blepharitis is inflammation of the eye-lids. See Table 14-3.

During an examination of a patient's abdomen, the nurse notes that the abdomen is rounded and firm to the touch. During percussion, the nurse notes a drum-like quality of the sound across the quadrants. This type of sound indicates: a. constipation. b. air-filled areas. c. the presence of a tumor. d. the presence of dense organs.

B A musical or drum-like sound (tympany) is the sound heard when percussion occurs over an air-filled viscus, such as the stomach or intestines.

31. A patient with pleuritis has been admitted to the hospital and complains of pain with breathing. What other key assessment finding would the nurse expect to find upon auscultation? a. Stridor b. Friction rub c. Crackles d. Wheezing

B A patient with pleuritis will exhibit a pleural friction rub upon auscultation. This is the sound made when pleurae become inflamed and rub together during respiration. The sound is superficial, coarse, and low-pitched, as if two pieces of leather are being rubbed together. Stridor is associated with croup, acute epiglottitis in children, and foreign body inhalation. Crackles are associated with several diseases, such as pneumonia, heart failure, chronic bronchitis, and others (see Table 18-6). Wheezes are associated with diffuse airway obstruction caused by acute asthma or chronic emphysema.

During an oral examination of a 4-year-old Native American child, the nurse notices that her uvula is partially split. Which of these statements is accurate? This is: a. a cleft palate and is common in Native Americans. b. a bifid uvula, which occurs in some Native American groups. c. due to an injury and should be reported to the authorities. d. palatinus, which occurs frequently in Native Americans.

B Bifid uvula, a condition in which the uvula is split either completely or partially, occurs in some Native American groups.

The nurse would use bimanual palpation technique in which situation? a. Palpating the thorax of an infant b. Palpating the kidneys and uterus c. Assessing pulsations and vibrations d. Assessing the presence of tenderness and pain

B Bimanual palpation requires the use of both hands to envelop or capture certain body parts or organs such as the kidneys, uterus, or adnexa. The other situations are not appropriate for bimanual palpation.

A patient has a positive Homans' sign. The nurse knows that a positive Homans' sign may indicate: a. venous insufficiency. b. deep vein thrombosis. c. severe edema. d. problems with arterial circulation.

B Calf pain on dorsiflexion of the foot is a positive Homans' sign, which occurs in about 35% of deep vein thromboses. It also occurs with superficial phlebitis, Achilles tendinitis, and gastrocnemius and plantar muscle injury.

A 40-year-old patient who has just finished chemotherapy for breast cancer tells the nurse that she is concerned about her mouth. During the assessment the nurse finds areas of buccal mucosa that are raw and red with some bleeding as well as other areas that have a white, cheesy coating. The nurse recognizes that this abnormality is: a. aphthous ulcers. b. candidiasis. c. leukoplakia. d. Koplik spots.

B Candidiasis is a white, cheesy, curd-like patch on the buccal mucosa and tongue. It scrapes off, leaving raw, red surface that bleeds easily. It also occurs after the use of antibiotics or corticosteroids and in immunosuppressed persons. See Table 16-4 for descriptions of the other lesions.

During an assessment of an infant, the nurse notes that the fontanels are depressed and sunken. The nurse suspects which condition? a. Rickets b. Dehydration c. Mental retardation d. Increased intracranial pressure

B Depressed and sunken fontanels occur with dehydration or malnutrition. Mental retardation and rickets have no effect on fontanels. Increased intracranial pressure would cause tense or bulging, and possibly pulsating fontanels.

The nurse is preparing to auscultate for heart sounds. Which technique is correct? a. Listen to the sounds at the aortic, tricuspid, pulmonic, and mitral areas. b. Listen by inching the stethoscope in a rough Z pattern, from the base of the heart across and down, then over to the apex. c. Listen to the sounds only at the site where the apical pulse is felt to be the strongest. d. Listen for all possible sounds at a time at each specified area.

B Do not limit auscultation of breath sounds to only four locations. Sounds produced by the valves may be heard all over the precordium. Inch the stethoscope in a rough Z pattern from the base of the heart across and down, then over to the apex. Or, start at the apex and work your way up. See Figure 19-22. Listen selectively to one sound at a time.

A mother brings her newborn in for an assessment and asks, "Is there something wrong with my baby? His head seems so big." The nurse recognizes that which statement is true regarding the relative proportions of the head and trunk of the newborn? a. At birth, the head is one fifth the total length. b. Head circumference should be greater than chest circumference at birth. c. The head size reaches 90% of its final size when the child is 3 years old. d. When the anterior fontanel closes at 2 months, the head will be more proportioned to the body.

B During the fetal period, head growth predominates. Head size is greater than chest circumference at birth, and the head size grows during childhood, reaching 90% of its final size when the child is age 6 years.

The nurse is examining an infant and prepares to elicit the Moro reflex at which time during the examination? a. When the infant is sleeping b. At the end of the examination c. Before auscultation of the thorax d. Halfway through the examination

B Elicit the Moro or "startle" reflex at the end of the examination because it may cause the infant to cry.

A patient is unable to differentiate between sharp and dull stimulation to both sides of her face. The nurse suspects: a. Bell's palsy. b. damage to the trigeminal nerve. c. frostbite with resultant paresthesia to the cheeks. d. scleroderma.

B Facial sensations of pain or touch are mediated by cranial nerve (CN) V, which is the trigeminal nerve. Bell's palsy is associated with CN VII damage. Frostbite and scleroderma are not associated this problem

During a physical education class, a student is hit in the eye with the end of a baseball bat. When examined in the emergency department, the nurse notices the presence of blood in the anterior chamber of the eye. This finding indicates the presence of: a. hypopyon. b. hyphema. c. corneal abrasion. d. Pterygium

B Hyphema is the term for blood in anterior chamber is a serious result of blunt trauma (a fist or a baseball) or spontaneous hemorrhage and may indicate scleral rupture or major intraocular trauma. See Table 14-7 for descriptions of the other terms.

When assessing a patient the nurse notes that the left femoral pulse as diminished, 1+/4+. What should the nurse do next? a. Document the finding. b. Auscultate the site for a bruit. c. Check for calf pain. d. Check capillary refill in the toes.

B If a pulse is weak or diminished at the femoral site, the nurse should auscultate for a bruit. Presence of a bruit, or turbulent blood flow, indicates partial occlusion. The other responses are not correct.

In assessing the carotid arteries of an older patient with cardiovascular disease, the nurse would: a. palpate the artery in the upper one third of the neck. b. listen with the bell of the stethoscope to assess for bruits. c. palpate both arteries simultaneously to compare amplitude. d. instruct patient to take slow deep breaths during auscultation.

B If cardiovascular disease is suspected, then the nurse should auscultate each carotid artery for the presence of a bruit. The nurse should avoid compressing the artery because this could create an artificial bruit, and it could compromise circulation if the carotid artery is already narrowed by atherosclerosis. Avoid excessive pressure on the carotid sinus area higher in the neck; excessive vagal stimulation here could slow down the heart rate, especially in older adults. Palpate only one carotid artery at a time to avoid compromising arterial blood to the b

When examining the mouth of an elderly patient, the nurse recognizes that which finding is due to the aging process? a. Teeth that appear shorter b. A tongue that looks smoother in appearance c. Buccal mucosa that is beefy red in appearance d. A small, painless lump on the dorsum of the tongue

B In the aging adult, the tongue looks smoother because of papillary atrophy. The teeth are slightly yellowed and appear longer because of recession of gingival margins

During an assessment the nurse has elevated a patient's legs 12 inches off the table and has had him wag his feet to drain off venous blood. After helping him to sit up and dangle his legs over the side of the table, the nurse should expect a normal finding at this point would be: a. marked elevational pallor. b. venous filling within 15 seconds. c. no change in coloration of the skin. d. color returning to the feet within 20 seconds of assuming a sitting position.

B In this test it normally takes 10 seconds or less for the color to return to the feet and 15 seconds for the veins of the feet to fill. Marked elevational pallor as well as delayed venous filling occurs with arterial insufficiency.

The nurse is using an otoscope to assess the nasal cavity. Which of these techniques is correct? a. Insert the speculum at least 3 cm into the vestibule. b. Avoid touching the nasal septum with the speculum. c. Gently displace the nose to the side that is being examined. d. Keep the speculum tip medial to avoid touching the floor of the nares.

B Insert the apparatus into the nasal vestibule, avoiding pressure on the sensitive nasal septum. The tip of the nose should be lifted up before inserting the speculum.

The nurse is examining the lymphatic system of a healthy 3-year-old child. Which finding should the nurse expect? a. Excessive swelling of the lymph nodes b. The presence of palpable lymph nodes c. No nodes palpable because of the immature immune system of a child d. Fewer numbers and a smaller size of lymph nodes compared with those of an adult

B Lymph nodes are relatively large in children, and the superficial ones often are palpable even when the child is healthy.

During an assessment, the nurse notices that a patient's left arm is swollen from the shoulder down to the fingers, with nonpitting brawny edema. The right arm is normal. The patient had a left-sided mastectomy 1 year ago. The nurse suspects which problem? a. Venous stasis b. Lymphedema c. Arteriosclerosis d. Deep vein thrombosis

B Lymphedema after breast cancer causes unilateral swelling and nonpitting brawny edema, with overlying skin indurated. It is caused by the removal of lymph nodes with breast surgery or damage to lymph nodes and channels with radiation therapy for breast cancer, and it can impede drainage of lymph. The other responses are not correct.

The nurse has just completed a lymph node assessment on a 60-year-old healthy female patient. The nurse knows that most lymph nodes in healthy adults are normally: a. shotty. b. not palpable. c. large, firm, and fixed to the tissue. d. rubbery, discrete, and mobile.

B Most lymph nodes are not palpable in adults. The palpability of lymph nodes decreases with age. Normal nodes feel movable, discrete, soft, and nontender.

A woman who is in the second trimester of pregnancy mentions that she has had "more nosebleeds than ever" since she became pregnant. The nurse recognizes that this is due to: a. a problem with the patient's coagulation system. b. increased vascularity in the upper respiratory tract as a result of the pregnancy. c. increased susceptibility to colds and nasal irritation. d. inappropriate use of nasal sprays.

B Nasal stuffiness and epistaxis may occur during pregnancy as a result of increased vascularity in the upper respiratory tract.

A patient says that she has recently noticed a lump in the front of her neck below her "Adam's apple" that seems to be getting bigger. During the assessment, the finding that leads the nurse to suspect that this may not be a cancerous thyroid nodule is that the lump (nodule): a. is tender. b. is mobile and not hard. c. disappears when the patient smiles. d. is hard and fixed to the surrounding structures.

B Painless, rapidly growing nodules may be cancerous, especially the appearance of a single nodule in a young person. However, cancerous nodules tend to be hard and fixed to surrounding structures, not mobile.

A patient has suddenly developed shortness of breath and appears to be in significant respiratory distress. After putting a call in to the physician and placing the patient on oxygen, which of these is the best action for the nurse to take when assessing the patient further? a. Count the patient's respirations. b. Percuss the thorax bilaterally, noting any differences in percussion tones. c. Call for a chest x-ray and wait for the results before beginning an assessment. d. Inspect the thorax for any new masses and bleeding associated with respirations.

B Percussion is always available, portable, and gives instant feedback regarding changes in underlying tissue density, which may yield clues of the patient's physical status.

The nurse is performing an eye assessment on an 80-year-old patient. Which of these findings is considered abnormal? a. A decrease in tear production b. Unequal pupillary constriction in response to light c. The presence of arcus senilis seen around the cornea d. Loss of the outer hair on the eyebrows due to a decrease in hair follicles

B Pupils are small in old age, and the pupillary light reflex may be slowed, but pupillary con-striction should be symmetric. The assessment findings in the other responses are considered normal in older persons.

The direction of blood flow through the heart is best described by which of these? a. Vena cava - right atrium - right ventricle - lungs - pulmonary artery - left atrium - left ventricle b. Right atrium - right ventricle - pulmonary artery - lungs - pulmonary vein - left atrium - left ventricle c. Aorta - right atrium - right ventricle - lungs - pulmonary vein - left atrium - left ventricle - vena cava d. Right atrium - right ventricle - pulmonary vein - lungs - pulmonary artery - left atrium - left ventricle

B Returning blood from the body empties into the right atrium and flows into the right ventricle and then goes to the lungs through the pulmonary artery. The lungs oxygenate the blood and it is then returned to the left atrium by the pulmonary vein. It goes from there to the left ventricle and then out to the body through the aorta.

The nurse keeps in mind that the most important reason to share information and offer brief teaching while performing the physical examination is to help: a. the examiner feel more comfortable and gain control of the situation. b. build rapport and increase the patient's confidence in the examiner. c. the patient understand his or her disease process and treatment modalities. d. the patient identify questions about his or her disease and potential areas of patient education.

B Sharing of information builds rapport and increases the patient's confidence in the examiner. It also gives the patient a little more control in a situation in which it is easy to feel completely helpless.

The component of the conduction system referred to as the pacemaker of the heart is the: a. atrioventricular (AV) node. b. sinoatrial (SA) node. c. bundle of His. d. bundle branches.

B Specialized cells in the SA node near the superior vena cava initiate an electrical impulse. Because the SA node has an intrinsic rhythm, it is the "pacemaker."

During a clinic visit, a woman in her seventh month of pregnancy complains that her legs feel "heavy in the calf" and that she often has foot cramps at night. The nurse notices that the patient has dilated, tortuous veins in her lower legs. Which condition is reflected by these findings? a. Deep vein thrombophlebitis b. Varicose veins c. Lymphedema d. Raynaud's phenomenon

B Superficial varicose veins are caused by incompetent distant valves on veins, which results in reflux of blood and producing dilated, tortuous veins. They are more common in women, and pregnancy can also be a cause. Symptoms include aching, heaviness in the calf, easy fatigability, and night leg or foot cramps. Dilated, tortuous veins are seen on assessment. See Table 20-5 for the description of deep vein thrombophlebitis. See Table 20-2 for descriptions of Raynaud's phenomenon and lymphedema.

A patient has come in for an examination and states, "I have this spot in front of my ear lobe here on my cheek that seems to be getting bigger and is tender. What do you think it is?" The nurse notes swelling below the angle of the jaw and suspects that it could be an inflammation of his: a. thyroid gland. b. parotid gland. c. occipital lymph node. d. submental lymph node.

B Swelling with the parotid gland occurs below the angle of the jaw and is most visible when the head is extended. Painful inflammation occurs with mumps, and swelling also occurs with abscesses or tumors. Swelling occurs anterior to the lower ear lobe.

In assessing for an S4 heart sound with a stethoscope, the nurse would listen with the: a. bell at the base with the patient leaning forward. b. bell at the apex with the patient in the left lateral position. c. diaphragm in the aortic area with the patient sitting. d. diaphragm in the pulmonic area with the patient supine.

B The S4 is a ventricular filling sound. It occurs when atria contract late in diastole. It is heard immediately before S1. This is a very soft sound with a very low pitch. The nurse needs a good bell and must listen for it. It is heard best at the apex, with the person in the left lateral position.

Which statement about the apices of the lungs is true? The apices of the lungs: a. are at the level of the second rib anteriorly. b. extend 3 to 4 cm above the inner third of the clavicles. c. are located at the sixth rib anteriorly and the eighth rib laterally. d. rest on the diaphragm at the fifth intercostal space in the midclavicular line.

B The apex of the lung on the anterior chest is 3 to 4 cm above the inner third of the clavicles. On the posterior chest, the apices are at the level of C7.

During an assessment, a patient tells the nurse that her fingers often change color when she goes out in cold weather. She describes these episodes as her fingers first turning white, then blue, then red with a burning, throbbing pain. The nurse suspects that she is experiencing: a. lymphedema. b. Raynaud's disease. c. deep vein thrombosis. d. chronic arterial insufficiency.

B The condition with episodes of abrupt, progressive tricolor change of the fingers in response to cold, vibration, or stress is known as Raynaud's disease.

The nurse knows that a normal finding when assessing the respiratory system of an elderly adult is: a. increased thoracic expansion. b. decreased mobility of the thorax. c. a decreased anteroposterior diameter. d. bronchovesicular breath sounds throughout the lungs.

B The costal cartilages become calcified with aging, resulting in a less mobile thorax. Chest expansion may be somewhat decreased, and the chest cage commonly shows an increased anteroposterior diameter

The nurse is teaching a review class on the lymphatic system. A participant shows correct understanding of the material with which statement? a. "Lymph flow is propelled by the contraction of the heart." b. "The flow of lymph is slow compared with that of the blood." c. "One of the functions of the lymph is to absorb lipids from the biliary tract." d. "Lymph vessels have no valves, so there is a free flow of lymph fluid from the tissue spaces into the bloodstream."

B The flow of lymph is slow compared with that of the blood. Lymph flow is not propelled by the heart, but rather by contracting skeletal muscles, pressure changes secondary to breathing, and by contraction of the vessel walls. Lymph does not absorb lipids from the biliary tract. The vessels do have valves, so flow is one way from the tissue spaces to the bloodstream.

The nurse is conducting a visual examination. Which of these statements regarding visual pathways and visual fields is true? a. The right side of the brain interprets vision for the right eye. b. The image formed on the retina is upside down and reversed from its actual appearance in the outside world. c. Light rays are refracted through the transparent media of the eye before striking the pupil. d. The light impulses are conducted through the optic nerve to the temporal lobes of the brain.

B The image formed on the retina is upside down and reversed from its actual appearance in the outside world. The light rays are refracted through the transparent media of the eye before striking the retina, and the nerve impulses are conducted through the optic nerve tract to the visual cortex of the occipital lobe of the brain. The left side of the brain interprets vision for the right eye.

A 35-year-old man is seen in the clinic for an infection in his left foot. Which of these findings should the nurse expect to see during an assessment of this patient? a. Hard and fixed cervical nodes b. Enlarged and tender inguinal nodes c. Bilateral enlargement of the popliteal nodes d. "Pellet-like" nodes in the supraclavicular region

B The inguinal nodes in the groin drain most of the lymph of the lower extremities. With local inflammation, the nodes in that area become swollen and tender.

When considering the biocultural differences in the respiratory systems, the nurse knows that which statement is true? a. The smallest chest volumes are found in Asians. b. The largest chest volumes are found in whites. c. Asians are most likely to develop asthma. d. Racial differences are of no significance when assessing the respiratory system.

B The largest chest volumes are found, in descending order, in whites, then African-Americans, Asians, and Native Americans. Even when the shorter height of Asians is considered, their chest volume remains significantly lower than that of whites and blacks. A disproportionately large number of tuberculosis cases are reported among blacks, most of whom were born in the United States.

The nurse is reviewing for a class in age-related changes in the eye. Which of these physiological changes is responsible for presbyopia? a. Degeneration of the cornea b. Loss of lens elasticity c. Decreased adaptation to darkness d. Decreased distance vision abilities

B The lens loses elasticity and decreases its ability to change shape to accommodate for near vision. This condition is called presbyopia.

A patient comes to the clinic complaining of neck and shoulder pain and is unable to turn her head. The nurse suspects damage to cranial nerve (CN) _____ and proceeds with the examination by _____. a. XI; palpating the anterior and posterior triangles b. XI; asking the patient to shrug her shoulders against resistance c. XII; percussing the sternomastoid and submandibular neck muscles d. XII; assessing for a positive Romberg sign

B The major neck muscles are the sternomastoid and the trapezius. They are innervated by CN XI, the spinal accessory. The innervated muscles assist with head rotation and head flexion, movement of the shoulders, and extension and turning of the head.

The primary purpose of the ciliated mucous membrane in the nose is to: a. warm the inhaled air. b. filter out dust and bacteria. c. filter coarse particles from inhaled air. d. facilitate movement of air through the nares.

B The nasal hairs filter the coarsest matter from inhaled air, whereas the mucous blanket filters out dust and bacteria. The rich blood supply of the nasal mucosa warms the inhaled air.

During an assessment, the nurse uses the "profile sign" to detect: a. pitting edema. b. early clubbing. c. symmetry of the fingers. d. insufficient capillary refill.

B The nurse should use the profile sign (viewing the finger from the side) to detect early clubbing.

A male patient with a history of AIDS has come in for an examination and he states, "I think that I have the mumps." The nurse would begin by examining the: a. thyroid gland. b. parotid gland. c. cervical lymph nodes. d. mouth and skin for lesions.

B The parotid gland may become swollen with the onset of mumps, and parotid enlargement has been found with HIV

The nurse is palpating the sinus areas. If the findings are normal, then the patient should report which sensation? a. No sensation b. Firm pressure c. Pain during palpation d. Pain sensation behind eyes

B The person should feel firm pressure but no pain. Sinus areas are tender to palpation in persons with chronic allergies and acute infection (sinusitis).

During an examination of the anterior thorax, the nurse keeps in mind that the trachea bifurcates anteriorly at the: a. costal angle. b. sternal angle. c. xiphoid process. d. suprasternal notch.

B The sternal angle marks the site of tracheal bifurcation into the right and left main bronchi; it corresponds with the upper border of the atria of the heart, and it lies above the fourth thoracic vertebra on the back.

In assessing the tonsils of a 30 year old, the nurse notices that they are involuted, granular in appearance, and appear to have deep crypts. What is correct response to these findings? a. Refer the patient to a throat specialist. b. Nothing, because this is the appearance of normal tonsils. c. Continue with assessment looking for any other abnormal findings. d. Obtain a throat culture on the patient for possible strep infection.

B The tonsils are the same color as the surrounding mucous membrane, although they look more granular and their surface shows deep crypts. Tonsillar tissue enlarges during childhood until puberty and then involutes

Which of these assessment findings would the nurse expect to see when examining the eyes of a black patient? a. Increased night vision b. A dark retinal background c. Increased photosensitivity d. Narrowed palpebral fissures

B There is an ethnically based variability in the color of the iris and in retinal pigmentation, with darker irides having darker retinas behind them.

The nurse is attempting to assess the femoral pulse in an obese patient. Which of these actions would be most appropriate? a. Have the patient assume a prone position. b. Ask the patient to bend his or her knees to the side in a froglike position. c. Press firmly against the bone with the patient in a semi-Fowler position. d. Listen with a stethoscope for pulsations because it is very difficult to palpate the pulse in an obese person.

B To help expose the femoral area, particularly in obese people, the nurse should ask the person to bend his or her knees to the side in a froglike position.

During an assessment of an adult, the nurse has noted unequal chest expansion and recognizes that this occurs in which situation? a. An obese patient b. When part of the lung is obstructed or collapsed c. When bulging of the intercostal spaces is present d. When accessory muscles are used to augment respiratory effort

B Unequal chest expansion occurs when part of the lung is obstructed or collapsed, as with pneumonia, or when guarding to avoid postoperative incisional pain or atelectasis.

The nurse is reviewing an assessment of a patient's peripheral pulses and notices that the documentation states that the radial pulses are "2+." The nurse recognizes that this reading indicates what type of pulse? a. Bounding b. Normal c. Weak d. Absent

B When documenting the force, or amplitude, of pulses, 3 indicates an increased, full, or bounding pulse, 2+ indicates a normal pulse, 1+ indicates a weak pulse, and 0 indicates an absent pulse.

The nurse is assessing a patient with possible cardiomyopathy and assesses the hepatojugular reflux. If heart failure is present, then the nurse should see which finding while pushing on the right upper quadrant of the patient's abdomen, just below the rib cage? a. The jugular veins will rise for a few seconds and then recede back to the previous level if the heart is working properly. b. The jugular veins will remain elevated as long as pressure on the abdomen is maintained. c. An impulse will be visible at the fourth or fifth intercostal space, at or inside the midclavicular line. d. The jugular veins will not be detected during this maneuver.

B When performing hepatojugular reflux, the jugular veins will rise for a few seconds and then recede back to the previous level if the heart is able to pump the additional volume created by the pushing; however, with heart failure, the jugular veins remain elevated as long as pressure on the abdomen is maintained.

During the history, a patient tells the nurse that he has frequent nosebleeds and asks the best way to get them to stop. What would be the nurse's best response? a. "While sitting up, place a cold compress over your nose." b. "Sit up with your head tilted forward and pinch your nose." c. "Just let the bleeding stop on its own, but don't blow your nose." d. "Lie on your back with your head tilted back and pinch your nose."

B With a nosebleed, the person should sit up with the head tilted forward and pinch the nose between the thumb and forefinger for 5 to 15 minutes

A teenage patient comes to the emergency department with complaints of an inability to breathe and a sharp pain in the left side of his chest. The assessment findings include cyanosis, tachypnea, tracheal deviation to the right, decreased tactile fremitus on the left, hyperresonance on the left, and decreased breath sounds on the left. The nurse interprets that these assessment findings are consistent with: a. bronchitis. b. a pneumothorax. c. acute pneumonia. d. an asthmatic attack.

B With a pneumothorax, free air in the pleural space causes partial or complete lung collapse. If the pneumothorax is large, then tachypnea and cyanosis are seen. Unequal chest expansion, decreased or absent tactile fremitus, tracheal deviation to the unaffected side, decreased chest expansion, hyperresonant percussion tones, and decreased or absent breath sounds are found with the presence of pneumothorax. See Table 18-8 for descriptions of the other conditions.

In assessing a 70-year-old man, the nurse finds the following: blood pressure 140/100 mm Hg; heart rate 104 and slightly irregular; split S2. Which of these findings can be explained by expected hemodynamic changes related to age? a. Increase in resting heart rate b. Increase in systolic blood pressure c. Decrease in diastolic blood pressure d. Increase in diastolic blood pressure

B With aging, there is an increase in systolic blood pressure. No significant change in diastolic pressure occurs with age. No change in resting heart rate occurs with aging. Cardiac output at rest is not changed with aging.

The nurse is preparing to examine a 4-year-old child. Which action is appropriate for this age group? a. Explain procedures in detail to alleviate the child's anxiety. b. Give the child feedback and reassurance during the examination. c. Do not ask the child to remove his clothes because children at this age are usually very private. d. Perform an examination of the ear, nose, and throat first, and then examine the thorax and abdomen.

B With preschool children use short, simple explanations. Children at this age are usually willing to undress. Examination of the head should be performed last. During the examination give the preschooler needed feedback and reassurance.

The nurse is presenting a class on risk factors for cardiovascular disease. Which of these are considered modifiable risk factors for myocardial infarction (MI)? Select all that apply. a. Ethnicity b. Abnormal lipids c. Smoking d. Gender e. Hypertension f. Diabetes g. Family history

B, C, E, F Nine modifiable risk factors for MI, as identified by a recent study, include abnormal lipids, smoking, hypertension, diabetes, abdominal obesity, psychosocial factors, consumption of fruits and vegetables, alcohol use, and regular physical activity

The nurse is teaching a health class to high-school-age boys. When discussing the topic of the use of smokeless tobacco (SLT), which of these statements are accurate? Select all that apply. a. One pinch of SLT in the mouth for 30 minutes delivers the equivalent of one cigarette. b. The use of SLT has been associated with a greater risk of oral cancer than smoking has. c. Pain is an early sign of oral cancer. d. Pain is rarely an early sign of oral cancer. e. Tooth decay is another risk of SLT because of the use of sugar as a sweetener. f. SLT is considered a healthy alternative to smoking.

B, D, E One pinch of SLT in the mouth for 30 minutes delivers the equivalent of three cigarettes; pain is rarely an early sign of oral cancer. Many brands of SLT are sweetened with sugars, which promotes tooth decay. SLT is not considered a healthy alternative to smoking, and the use of SLT has been associated with a greater risk of oral cancer than smoking has.

During an assessment, a patient mentions that "I just can't smell like I used to. I can barely smell the roses in my garden. Why is that?" The nurse will assess for what possible causes of changes in the sense of smell? Select all that apply. a. Chronic alcohol use b. Cigarette smoking c. Frequent episodes of strep throat d. Chronic allergies e. Aging f. Herpes simplex I

B, D, E The sense of smell diminishes with cigarette smoking, chronic allergies, and aging. Chronic alcohol use, history of strep throat, and herpes simplex I are not associated with changes in the sense of smell

The nurse is assessing a 1-month-old infant at his well-baby check up. Which assessment findings are appropriate for this age? Select all that apply. a. Head circumference equal to chest circumference b. Head circumference greater than chest circumference c. Head circumference less than chest circumference d. Fontanels firm and slightly concave e. Absent tonic neck reflex f. Cervical lymph nodes not palpable

B, D, F An infant's head circumference is larger than the chest circumference. At age 2, both measurements are the same. During childhood the chest circumference grows to exceed head circumference by 5 to 7 cm. Fontanels should feel firm and slightly concave in the infant, and they should close by age 9 months. Tonic neck reflex is present until between 3 and 4 months of age, and cervical lymph nodes are normally not palpable in an infant.

During an examination, a patient states that she was diagnosed with open-angle glaucoma 2 years ago. The nurse assesses for characteristics of open-angle glaucoma. Which of these are characteristics of open-angle glaucoma? Select all that apply. a. The patient may experience sensitivity to light, nausea, and halos around lights. b. The patient experiences tunnel vision in late stages. c. Immediate treatment is needed. d. Vision loss begins with peripheral vision. e. It causes sudden attacks of increased pressure that cause blurred vision. f. There are virtually no symptoms.

B, D, F Open-angle glaucoma is the most common type of glaucoma; there are virtually no symptoms. Vision loss begins with the peripheral vision, which often goes unnoticed because individuals learn to compensate intuitively by turning their heads. The other characteristics are those of closed-angle glaucoma.

The nurse notices that an infant has a large, soft lump on the side of his head and that his mother is very concerned. She tells the nurse that she noticed the lump about 8 hours after her baby's birth, and that it seems to be getting bigger. One possible explanation for this is: a. hydrocephalus. b. craniosynostosis. c. cephalhematoma. d. caput succedaneum.

C A cephalhematoma is a subperiosteal hemorrhage that is the result of birth trauma. It is soft, fluctuant, and well defined over one cranial bone. It appears several hours after birth and gradually increases in size.

The nurse is assessing the pulses of a patient who has been admitted for untreated hyperthyroidism. The nurse should expect to find a(n) _____ pulse. a. normal b. absent c. bounding d. weak, thready

C A full, bounding pulse occurs with hyperkinetic states (such as exercise, anxiety, fever), anemia, and hyperthyroidism. Absent pulse occurs with occlusion. Weak, thready pulses occur with shock and peripheral artery disease.

The nurse is preparing to perform a modified Allen test. Which is an appropriate reason for this test? a. To measure the rate of lymphatic drainage b. To evaluate the adequacy of capillary patency before venous blood draws c. To evaluate the adequacy of collateral circulation before cannulating the radial artery d. To evaluate the venous refill rate that occurs after the ulnar and radial arteries are temporarily occluded

C A modified Allen test is used to evaluate the adequacy of collateral circulation before

The nurse is describing a weak, thready pulse on the documentation flow sheet. Which statement is correct? a. "Easily palpable, pounds under the fingertips." b. "Greater than normal force, then collapses suddenly." c. "Hard to palpate, may fade in and out, easily obliterated by pressure." d. "Rhythm is regular, but force varies with alternating beats of large and small amplitude."

C A weak, thready pulse is hard to palpate, may fade in and out, and is easily obliterated by pressure. It is associated with decreased cardiac output and peripheral arterial disease.

During an admission assessment, the nurse notices that a male patient has an enlarged and rather thick skull. The nurse suspects acromegaly and would further assess for: a. exophthalmos. b. bowed long bones. c. coarse facial features. d. an acorn-shaped cranium.

C Acromegaly is excessive secretion of growth hormone that creates an enlarged skull and thickened cranial bones. Patients will have elongated heads, massive faces, prominent noses and lower jaws, heavy eyebrow ridges, and coarse facial features. Exophthalmos is associated with hyperthyroidism. Bowed long bones and an acorn-shaped cranium result from Paget's disease.

During a cardiovascular assessment, the nurse knows that an S4 heart sound is: a. heard at the onset of atrial diastole. b. usually a normal finding in the elderly. c. heard at the end of ventricular diastole. d. heard best over the second left intercostal space with the individual sitting upright.

C An S4 heart sound is heard at the end of diastole when the atria contract (atrial systole) and when the ventricles are resistant to filling. The S4 occurs just before the S1.

When assessing the respiratory system of a 4-year-old child, which of these findings would the nurse expect? a. Crepitus palpated at the costochondral junctions b. No diaphragmatic excursion as a result of a child's decreased inspiratory volume c. The presence of bronchovesicular breath sounds in the peripheral lung fields d. An irregular respiratory pattern and a respiratory rate of 40 breaths per minute at rest

C Bronchovesicular breath sounds in the peripheral lung fields of the infant and young child up to age 5 or 6 years are a normal finding. Their thin chest walls with underdeveloped musculature do not dampen the sound, as do the thicker chest walls of adults, so breath sounds are louder and harsher.

The nurse is reviewing the characteristics of breath sounds. Which statement about bronchovesicular breath sounds is true? They are: a. musical in quality. b. usually pathological. c. expected near the major airways. d. similar to bronchial sounds except that they are shorter in duration.

C Bronchovesicular sounds are heard over major bronchi where fewer alveoli are located: posteriorly, between the scapulae, especially on the right; anteriorly, around the upper sternum in the first and second intercostal spaces. The other responses are not correct.

A mother brings in her newborn infant for an assessment and tells the nurse that she has noticed that whenever her newborn's head is turned to the right side, she straightens out the arm and leg on the same side and flexes the opposite arm and leg. After finding this on examination, the nurse would tell her that this is: a. abnormal and is called the atonic neck reflex. b. normal and should disappear by the first year of life. c. normal and should disappear between 3 and 4 months of age. d. abnormal. The baby should be flexing the arm and leg on the right side of his body when the head is turned to the right.

C By 2 weeks the infant shows the tonic neck reflex when supine and the head is turned to one side (extension of same arm and leg, flexion of opposite arm and leg). The tonic neck reflex disappears between 3 and 4 months of age.

The nurse is assessing a 3 year old for "drainage from the nose." On assessment, it is found that there is a purulent drainage from the left nares that has a very foul odor and no drainage from the right nares. The child is afebrile with no other symptoms. What should the nurse do next? a. Refer to the physician for an antibiotic order. b. Have the mother bring the child back in 1 week. c. Perform an otoscopic examination of the left nares. d. Tell the mother that this is normal for children of this age.

C Children are apt to put an object up the nose, producing unilateral purulent drainage and foul odor. Because some risk for aspiration exists, removal should be prompt

A woman in her 26th week of pregnancy states that she is "not really short of breath" but feels that she is aware of her breathing and the need to breathe. What is the nurse's best reply? a. "The diaphragm becomes fixed during pregnancy, making it difficult to take in a deep breath." b. "The increase in estrogen levels during pregnancy often causes a decrease in the diameter of the rib cage and makes it difficult to breathe." c. "What you are experiencing is normal. Some women may interpret this as shortness of breath, but it is a normal finding and nothing is wrong." d. "This is normal as the fetus grows because of the increased oxygen demand on the mother's body and results in an increased respiratory rate."

C During pregnancy, the woman may develop an increased awareness of the need to breathe. Some women may interpret this as dyspnea, even though structurally nothing is wrong. Estrogen increases relax the chest cage ligaments, causing an increase in transverse diameter. The growing fetus does increase the oxygen demand on the mother's body, but this is met easily by the increasing tidal volume (deeper breathing). Little change occurs in the respiratory rate.

A 92-year-old patient has had a stroke. The right side of his face is drooping. The nurse might also suspect which of these assessment findings? a. Epistaxis b. Rhinorrhea c. Dysphagia d. Xerostomia

C Dysphagia is difficulty with swallowing and may occur with a variety of disorders, including stroke and other neurologic diseases. Rhinorrhea is a runny nose; epistaxis is a bloody nose. Xerostomia is a dry mouth.

During an oral assessment of a 30-year-old African-American patient, the nurse notices bluish lips and a dark line along the gingival margin. What would the nurse do in response to this finding? a. Check the patient's hemoglobin for anemia. b. Assess for other signs of insufficient oxygen supply. c. Proceed with assessment, knowing that this is a normal finding. d. Ask if he has been exposed to an excessive amount of carbon monoxide.

C Some African-Americans normally may have bluish lips and a dark line on the gingival margin.

Which of these statements describes the closure of the valves in a normal cardiac cycle? a. The aortic valve closes slightly before the tricuspid valve. b. The pulmonic valve closes slightly before the aortic valve. c. The tricuspid valve closes slightly later than the mitral valve. d. Both the tricuspid and pulmonic valves close at the same time.

C Events occur just slightly later in the right side of the heart because of the route of myocardial depolarization. As a result, two distinct components to each of the heart sounds exist, and sometimes they can be heard separately. In the first heart sound, the mitral component (M1) closes just before the tricuspid component (T1).

A visitor from Poland who does not speak English seems to be somewhat apprehensive about the nurse examining his neck. He would probably be most comfortable with the nurse examining his thyroid from: a. behind with the nurse's hands placed firmly around his neck. b. the side with the nurse's eyes averted toward the ceiling and thumbs on his neck. c. the front with the nurse's thumbs placed on either side of his trachea and his head tilted forward. d. the front with the nurse's thumbs placed on either side of his trachea and his head tilted backward.

C Examining this patient's thyroid from the back may be unsettling for him. It would be best to examine his thyroid using the anterior approach, asking him to tip his head forward and to the right and then the left

A mother asks when her newborn infant's eyesight will be developed. The nurse should reply: a. "Vision is not totally developed until 2 years of age." b. "Infants develop the ability to focus on an object at around 8 months." c. "By about 3 months, infants develop more coordinated eye movements and can fixate on an object." d. "Most infants have uncoordinated eye movements for the first year of life."

C Eye movements may be poorly coordinated at birth, but by 3 to 4 months of age, the infant should establish binocularity and should be able to fixate on a single image with both eyes simultaneously.

The nurse notices that a patient's palpebral fissures are not symmetrical. On examination, the nurse may find that there has been damage to cranial nerve: a. III. b. V. c. VII. d. VIII.

C Facial muscles are mediated by cranial nerve (CN) VII; asymmetry of palpebral fissures may be due to CN VII damage (Bell's palsy).

A patient has been admitted to the emergency department with a possible medical diagnosis of pulmonary embolism. The nurse expects to see which assessment findings related to this condition? a. Absent or decreased breath sounds b. Productive cough with thin, frothy sputum c. Chest pain that is worse on deep inspiration, dyspnea d. Diffuse infiltrates with areas of dullness upon percussion

C Findings for pulmonary embolism include chest pain that is worse on deep inspiration, dyspnea, apprehension, anxiety, restlessness, PaO2 less than 80, diaphoresis, hypotension, crackles, and wheezes

When assessing a newborn infant who is 5 minutes old, the nurse knows that which of these statements would be true? a. The left ventricle is larger and weighs more than the right ventricle. b. The circulation of a newborn is identical to that of an adult. c. There is an opening in the atrial septum where blood can flow into the left side of the heart. d. The foramen ovale closes just minutes before birth and the ductus arteriosus closes immediately after.

C First, about two thirds of the blood is shunted through an opening in the atrial septum, the foramen ovale into the left side of the heart, where it is pumped out through the aorta. The foramen ovale closes within the first hour because the pressure in the right side of the heart is now lower than in the left side.

The nurse is reviewing the technique of palpating for tactile fremitus with a new graduate. Which statement by the graduate nurse reflects a correct understanding of tactile fremitus? "Tactile fremitus: a. is caused by moisture in the alveoli." b. indicates that there is air in the subcutaneous tissues." c. is caused by sounds generated from the larynx." d. reflects the blood flow through the pulmonary arteries."

C Fremitus is a palpable vibration. Sounds generated from the larynx are transmitted through patent bronchi and the lung parenchyma to the chest wall where they are felt as vibrations. Crepitus is the term for air in the subcutaneous tissues.

A patient's thyroid is enlarged, and the nurse is preparing to auscultate the thyroid for the presence of a bruit. A bruit is a __________ of the stethoscope. a. low gurgling sound best heard with the diaphragm b. loud, whooshing, blowing sound best heard with the bell c. soft, whooshing, pulsatile sound best heard with the bell d. high-pitched tinkling sound best heard with the diaphragm

C If the thyroid gland is enlarged, the nurse should auscultate it for the presence of a bruit, which is a soft, pulsatile, whooshing, blowing sound heard best with the bell of the stethoscope

The nurse is performing a cardiac assessment on a 65-year-old patient 3 days after her myocardial infarction. Heart sounds are normal when she is supine, but when she is sitting and leaning forward, the nurse hears a high-pitched, scratchy sound with the diaphragm of the stethoscope at the apex. It disappears on inspiration. The nurse suspects: a. increased cardiac output. b. another myocardial infarction. c. inflammation of the precordium. d. ventricular hypertrophy resulting from muscle damage.

C Inflammation of the precordium gives rise to a friction rub. The sound is high pitched and scratchy, like sandpaper being rubbed. It is best heard with the diaphragm of the stethoscope, with the person sitting up and leaning forward, and with the breath held in expiration. A friction rub can be heard any place on the precordium but usually is best heard at the apex and left lower sternal border, which are places where the pericardium comes in close contact with the chest wall.

A woman comes to the clinic and states, "I've been sick for so long! My eyes have gotten so puffy, and my eyebrows and hair have become coarse and dry." The nurse will assess for other signs and symptoms of: a. cachexia. b. Parkinson's syndrome. c. myxedema. d. scleroderma.

C Myxedema (hypothyroidism) is a deficiency of thyroid hormone that, when severe, causes a nonpitting edema or myxedema. The patient will have a puffy edematous face especially around eyes (periorbital edema), coarse facial features, dry skin, and dry, coarse hair and eyebrows. See Table 13-4, Abnormal Facial Appearances with Chronic Illnesses, for descriptions of the other responses.

The nurse is assessing an 80-year-old patient. Which of these findings would be expected for this patient? a. Hypertrophy of the gums b. An increased production of saliva c. A decreased ability to identify odors d. Finer and less prominent nasal hair

C The sense of smell may be reduced because of a decrease in the number of olfactory nerve fibers. Nasal hairs grow coarser and stiffer with aging. The gums may recede with aging, not hypertrophy, and there is a decrease in saliva production

The nurse is performing an assessment on an adult. The adult's vital signs are normal and capillary refill time is 5 seconds. What should the nurse do next? a. Ask the patient about a past history of frostbite. b. Suspect that the patient has a venous insufficiency problem. c. Consider this a delayed capillary refill time and investigate further. d. Consider this a normal capillary refill time that requires no further assessment.

C Normal capillary refill time is less than 1 to 2 seconds. The following conditions can skew the findings: a cool room, decreased body temperature, cigarette smoking, peripheral edema, and anemia.

During an assessment, the nurse knows that expected assessment findings in the normal adult lung include the presence of: a. adventitious sounds and limited chest expansion. b. increased tactile fremitus and dull percussion tones. c. muffled voice sounds and symmetrical tactile fremitus. d. absent voice sounds and hyperresonant percussion tones.

C Normal lung findings include symmetric chest expansion, resonant percussion tones, vesicular breath sounds over the peripheral lung fields, muffled voice sounds, and no adventitious sounds.

The nurse is performing a well-child check on a 5-year-old boy. He has no current history that would lead the nurse to suspect illness. His medical history is unremarkable, and he received immunizations 1 week ago. Which of these findings should be considered normal in this situation? a. Enlarged, warm, tender nodes b. Lymphadenopathy of the cervical nodes c. Palpable firm, small, shotty, mobile, nontender lymph nodes d. Firm, rubbery, large nodes, somewhat fixed to the underlying tissue

C Palpable lymph nodes are often normal in children and infants. They are small, firm, shotty, mobile, and nontender. Vaccinations can produce lymphadenopathy. Enlarged, warm, tender nodes indicate current infection

During the precordial assessment on an patient who is 8 months pregnant, the nurse palpates the apical impulse at the fourth left intercostal space lateral to the midclavicular line. This finding would indicate: a. right ventricular hypertrophy. b. increased volume and size of the heart as a result of pregnancy. c. displacement of the heart from elevation of the diaphragm. d. increased blood flow through the internal mammary artery.

C Palpation of the apical impulse is higher and more lateral compared with the normal position because the enlarging uterus elevates the diaphragm and displaces the heart up and to the left and rotates it on its long axis.

A 45-year-old man is in the clinic for a routine physical. During the history the patient states he's been having difficulty sleeping. "I'll be sleeping great and then I wake up and feel like I can't get my breath." The nurse's best response to this would be: a. "When was your last electrocardiogram?" b. "It's probably because it's been so hot at night." c. "Do you have any history of problems with your heart?" d. "Have you had a recent sinus infection or upper respiratory infection?"

C Paroxysmal nocturnal dyspnea occurs with heart failure. Lying down increases volume of intrathoracic blood, and the weakened heart cannot accommodate the increased load. Classically, the person awakens after 2 hours of sleep, arises, and flings open a window with the perception of needing fresh air

During auscultation of the lungs of an adult patient, the nurse notices the presence of bronchophony. The nurse should assess for signs of which condition? a. Airway obstruction b. Emphysema c. Pulmonary consolidation d. Asthma

C Pathologic conditions that increase lung density, such as pulmonary consolidation, will enhance transmission of voice sounds, such as bronchophony. See Table 18-7.

The nurse notices the presence of periorbital edema when performing an eye assessment on a 70-year-old patient. The nurse should: a. check for the presence of exophthalmos. b. suspect that the patient has hyperthyroidism. c. ask the patient if he or she has a history of heart failure. d. assess for blepharitis because this is often associated with periorbital edema.

C Periorbital edema occurs with local infections, crying, and systemic conditions such as heart failure, renal failure, allergy, and hypothyroidism. Periorbital edema is not associated with blepharitis.

During an assessment of an older adult, the nurse should expect to notice which finding as a normal physiologic change associated with the aging process? a. Hormonal changes causing vasodilation and a resulting drop in blood pressure b. Progressive atrophy of the intramuscular calf veins, causing venous insufficiency c. Peripheral blood vessels growing more rigid with age, producing a rise in systolic blood pressure d. Narrowing of the inferior vena cava, causing low blood flow and increases in venous pressure resulting in varicosities

C Peripheral blood vessels grow more rigid with age, resulting in a rise in systolic blood pressure. Aging produces progressive enlargement of the intramuscular calf veins, not atrophy. The other options are not correct.

A 60-year-old man is at the clinic for an eye examination. The nurse suspects that he has ptosis of one eye. How should the nurse check for this? a. Perform the confrontation test. b. Assess the individual's near vision. c. Observe the distance between the palpebral fissures. d. Perform the corneal light test and look for symmetry of the light reflex.

C Ptosis is drooping of the upper eyelid that would be apparent by observing the distance be-tween the upper and lower eyelids. The confrontation test measures peripheral vision. Measuring near vision or the corneal light test does not check for ptosis.

During the examination, it is often appropriate to offer some brief teaching about the patient's body or the examiner's findings. Which of these statements by the nurse is most appropriate? a. "Your atrial dysrhythmias are under control." b. "You have pitting edema and mild varicosities." c. "Your pulse is 80 beats per minute. This is within the normal range." d. "I'm using my stethoscope to listen for any crackles, wheezes, or rubs."

C Sharing of some information builds rapport as long as the patient is able to understand the terminology.

A patient comes into the clinic complaining of facial pain, fever, and malaise. On examination, the nurse notes swollen turbinates and purulent discharge from the nose. The patient also complains of a dull, throbbing pain in his cheeks and teeth on the right side and pain when the nurse palpates the areas. The nurse recognizes that this patient has: a. posterior epistaxis. b. frontal sinusitis. c. maxillary sinusitis. d. nasal polyps.

C Signs of maxillary sinusitis include facial pain, after upper respiratory infection, red swollen nasal mucosa, swollen turbinates, and purulent discharge. The person also has fever, chills, and malaise. With maxillary sinusitis, dull throbbing pain occurs in cheeks and teeth on the same side, and pain with palpation is present. With frontal sinusitis, pain is above the supraorbital ridge.

Which of these findings would the nurse expect to notice during a cardiac assessment on a 4-year-old child? a. S3 when sitting up b. Persistent tachycardia above 150 c. Murmur at second left intercostal space when supine d. Palpable apical impulse in fifth left intercostal space lateral to midclavicular line

C Some murmurs are common in healthy children or adolescents and are termed innocent or functional. The innocent murmur is heard at the second or third left intercostal space and disappears with sitting, and the young person has no associated signs of cardiac dysfunction.

A 35-year-old recent immigrant is being seen in the clinic for complaints of a cough that is associated with rust-colored sputum, low-grade afternoon fevers, and night sweats for the past 2 months. The nurse's preliminary analysis, based on this history, is that this patient may be suffering from: a. bronchitis. b. pneumonia. c. tuberculosis. d. pulmonary edema.

C Sputum is not diagnostic alone, but some conditions have characteristic sputum production. Tuberculosis often produces rust-colored sputum in addition to other symptoms of night sweats and low-grade afternoon fevers. See Table 18-8.

When examining a patient's eyes, the nurse recalls that stimulation of the sympathetic branch of the autonomic nervous system: a. causes pupillary constriction. b. adjusts the eye for near vision. c. elevates the eyelid and dilates the pupil. d. causes contraction of the ciliary body.

C Stimulation of the sympathetic branch of the autonomic nervous system dilates the pupil and elevates the eyelid. Parasympathetic nervous system stimulation causes the pupil to constrict. The muscle fibers of the iris contract the pupil in bright light to accommodate for near vision. The ciliary body controls the thickness of the lens.

The nurse is assessing color vision of a male child. Which statement is correct? The nurse should: a. check color vision annually until the age of 18 years. b. ask the child to identify the color of his or her clothing. c. test for color vision once between the ages of 4 and 8. d. begin color vision screening at the child's 2-year check-up.

C Test only boys for color vision once between the ages of 4 and 8 years. It is not tested in fe-males because it is rare in females. Testing is done with the Ishihara test, which is a series of polychromatic cards.

The nurse is preparing to assess the visual acuity of a 16-year-old patient. How should the nurse proceed? a. Perform the confrontation test. b. Ask the patient to read the print on a handheld Jaeger card. c. Use the Snellen chart positioned 20 feet away from the patient. d. Determine the patient's ability to read newsprint at a distance of 12 to 14 inches.

C The Snellen alphabet chart is the most commonly used and most accurate measure of visual acuity. The confrontation test is a gross measure of peripheral vision. The Jaeger card or newspaper tests are used to test near vision.

The nurse is assessing a patient's apical impulse. Which of these statements is true regarding the apical impulse? a. It is palpable in all adults. b. It occurs with the onset of diastole. c. Its location may be indicative of heart size. d. It should normally be palpable in the anterior axillary line.

C The apical impulse is palpable in about 50% of adults. It is located in the fifth left intercostal space in the midclavicular line. Horizontal or downward displacement of the apical impulse may indicate an enlargement of the left ventricle.

During an examination, the nurse finds that a patient's left temporal artery is tortuous and feels hardened and tender compared with the right temporal artery. The nurse suspects which condition? a. Crepitation b. Mastoiditis c. Temporal arteritis d. Bell palsy

C The artery looks more tortuous and feels hardened and tender with temporal arteritis. These assessment findings are not consistent with the other responses.

A 2-year-old child has been brought to the clinic for a well-child check-up. The best way for the nurse to begin the assessment is reflected by which statement? a. Ask the parent to place the child on the examining table. b. Have the parent remove all of the child's clothing before the examination. c. Allow the child to keep a security object such as a toy or blanket during the examination. d. Initially focus interactions on the child, essentially "ignoring" the parent, until the child's trust has been obtained.

C The best place to examine the toddler is on the parent's lap. Toddlers understand symbols, so a security object is helpful. Initially, focus more on the parent. This allows the child to gradually adjust and become familiar with you. A 2-year-old child does not like to take off his or her clothes. Have the parent undress one body part at a time.

During an assessment the nurse notices that an elderly patient has tears rolling down his face from his left eye. Closer examination shows that the lower lid is loose and rolling outward. The patient complains of his eye feeling "dry and itchy." Which action by the nurse is correct? a. Assess the eye for a possible foreign body. b. Document the finding as ptosis. c. Assess for other signs of ectropion. d. Contact the prescriber because these are signs of basal cell carcinoma.

C The condition described is known as ectropion, and it occurs in aging due to atrophy of elastic and fibrous tissues. The lower lid does not approximate to the eyeball, and, as a result, the puncta cannot siphon tears effectively, and excessive tearing results. Ptosis is drooping of the upper eyelid. These are not signs of a foreign body in the eye or basal cell carcinoma.

The nurse is auscultating the chest in an adult. Which technique is correct? a. Instruct the patient to take deep, rapid breaths. b. Instruct the patient to breathe in and out through his or her nose. c. Use the diaphragm of the stethoscope held firmly against the chest. d. Use the bell of the stethoscope held lightly against the chest to avoid friction.

C The diaphragm of the stethoscope held firmly on the chest is the correct way to auscultate breath sounds. The patient should be instructed to breathe through his or her mouth, a little deeper than usual, but not to hyperventilate.

A patient, an 85-year-old woman, is complaining about the fact that the bones in her face have become more noticeable. What explanation should the nurse give to her? a. Diets low in protein and high in carbohydrates may cause enhanced facial bones. b. It is probably because she doesn't use a dermatologically approved moisturizer. c. It is probably due to a combination of factors related to aging, such as decreased elasticity, subcutaneous fat, and moisture in her skin. d. Facial skin becomes more elastic with age. This increased elasticity causes the skin to be more taught, drawing attention to the facial bones.

C The facial bones and orbits appear more prominent in the aging adult, and the facial skin sags owing to decreased elasticity, decreased subcutaneous fat, and decreased moisture in the skin.

A mother is concerned because her 18-month-old child has 12 teeth. She is wondering if this is normal for a child of this age. The nurse's best response would be: a. "How many teeth did you have at this age?" b. "All 20 deciduous teeth are expected to erupt by age 4 years." c. "This is a normal number of teeth for an 18 month old." d. "Normally, by age 2 1/2, 16 deciduous teeth are expected."

C The guidelines for the number of teeth for children under 2 years old are as follows: the child's age in months minus the number 6 should be equal to the expected number of deciduous teeth. Normally, all 20 teeth are in by 2 1/2 years old. In this instance, the child is 18 months old, minus 6, equals 12 deciduous teeth expected

The projections in the nasal cavity that increase the surface area are called the: a. meatus. b. septum. c. turbinates. d. Kiesselbach plexus.

C The lateral walls of each nasal cavity contain three parallel bony projections: the superior, middle, and inferior turbinates. They increase the surface area so that more blood vessels and mucous membrane are available to warm, humidify, and filter the inhaled air.

The nurse is reviewing the blood supply to the arm. The major artery supplying the arm is the _____ artery. a. ulnar b. radial c. brachial d. deep palmar

C The major artery supplying the arm is the brachial artery. The brachial artery bifurcates into the ulnar and radial arteries immediately below the elbow. In the hand, the ulnar and radial arteries form two arches known as the superficial and deep palmar arches.

The most important step that the nurse can take to prevent transmission of microorganisms in the hospital setting is to: a. wear protective eye wear at all times. b. wear gloves during any and all contact with patients. c. wash hands before and after contact with each patient. d. clean the stethoscope with an alcohol swab between patients.

C The most important step to decrease the risk of microorganism transmission is to wash hands promptly and thoroughly before and after physical contact with each patient. Stethoscopes should also be cleansed with an alcohol swab before and after each patient contact. The best routine is to combine stethoscope rubbing with hand hygiene each time hand hygiene is performed.

When the nurse is auscultating the carotid artery for bruits, which of these statements reflects correct technique? a. While listening with the bell of the stethoscope, have the patient take a deep breath and hold it. b. While auscultating one side with the bell of the stethoscope, palpate the carotid artery on the other side to check pulsations. c. Lightly apply the bell of the stethoscope over the carotid artery, and while listening, have the patient take a breath, exhale, and hold it briefly. d. Firmly place the bell of the stethoscope over the carotid artery, and while listening, have the patient take a breath, exhale, and hold it briefly.

C The nurse should lightly apply the bell of the stethoscope over the carotid artery at three levels; while listening, the nurse should have the patient take a breath, exhale, and hold it briefly. Holding the breath on inhalation will also tense the levator scapulae muscles, which makes it hard to hear the carotids. Examine only one carotid artery at a time to avoid compromising arterial blood flow to the brain. Avoid pressure over the carotid sinus, which may lead to decreased heart rate, decreased blood pressure, and cerebral ischemia with syncope

The nurse notices that the mother of a 2-year-old boy brings him into the clinic quite frequently for various injuries and suspects there may be some child abuse involved. In doing the inspection of his mouth, the nurse should inspect for: a. swollen, red tonsils. b. ulcerations on the hard palate. c. bruising on the buccal mucosa or gums. d. small yellow papules along the hard palate.

C The nurse should notice any bruising or laceration on the buccal mucosa or gums of an infant or young child. Trauma may indicate child abuse from a forced feeding of a bottle or spoon.

A 65-year-old patient with a history of heart failure comes to the clinic with complaints of "being awakened from sleep with shortness of breath." Which action by the nurse is most appropriate? a. Obtain a detailed history of the patient's allergies and history of asthma. b. Tell the patient to sleep on his or her right side to facilitate ease of respirations. c. Assess for other signs and symptoms of paroxysmal nocturnal dyspnea. d. Assure the patient that this is normal and will probably resolve within the next week.

C The patient is experiencing paroxysmal nocturnal dyspnea: being awakened from sleep with shortness of breath and the need to be upright to achieve comfort

The nurse notes hyperresonant percussion tones when percussing the thorax of an infant. The nurse's best action would be to: a. notify the physician. b. suspect a pneumothorax. c. consider this a normal finding. d. monitor the infant's respiratory rate and rhythm.

C The percussion note of hyperresonance occurs normally in the infant and young child, owing to the relatively thin chest wall. Anything less than hyperresonance would have the same clinical significance as would dullness in the adult.

When a light is directed across the iris of a patient's eye from the temporal side, the nurse is assessing for: a. drainage from dacryocystitis. b. the presence of conjunctivitis over the iris. c. the presence of shadows, which may indicate glaucoma. d. a scattered light reflex, which may be indicative of cataracts.

C The presence of shadows in the anterior chamber may be a sign of acute angle-closure glau-coma. The normal iris is flat and creates no shadows. This is not the correct method for assessment for dacryocystitis, conjunctivitis, or cataracts.

In using the ophthalmoscope to assess a patient's eyes, the nurse notices a red glow in the patient's pupils. On the basis of this finding, the nurse would: a. suspect that there is an opacity in the lens or cornea. b. check the light source of the ophthalmoscope to verify that it is functioning. c. consider this a normal reflection of the ophthalmoscope light off the inner retina. d. continue with the ophthalmoscopic examination and refer the patient for further eval-uation.

C The red glow filling the person's pupil is the red reflex, and it is a normal finding caused by the reflection of the ophthalmoscope light off the inner retina. The other responses are not cor-rect.

When performing a respiratory assessment on a patient, the nurse notices a costal angle of approximately 90 degrees. This characteristic is: a. seen in patients with kyphosis. b. indicative of pectus excavatum. c. a normal finding in a healthy adult. d. an expected finding in a patient with a barrel chest.

C The right and left costal margins form an angle where they meet at the xiphoid process. Usually, this angle is 90 degrees or less. The angle increases when the rib cage is chronically overinflated, as in emphysema.

When assessing the pupillary light reflex, the nurse should use which technique? a. Shine a penlight from directly in front of the patient and inspect for pupillary constriction. b. Ask the patient to follow the penlight in eight directions and observe for bilateral pupil constriction. c. Shine a light across the pupil from the side and observe for direct and consensual pupillary constriction. d. Ask the patient to focus on a distant object. Then ask the patient to follow the pen-light to about 7 cm from the nose.

C To test the pupillary light reflex, the nurse should advance a light in from the side and note the direct and consensual pupillary constriction.

When examining the face, the nurse is aware that the two pairs of salivary glands that are accessible to examination are the _____ glands. a. occipital and submental b. parotid and jugulodigastric c. parotid and submandibular d. submandibular and occipital

C Two pairs of salivary glands accessible to examination on the face are the parotid glands, which are in the cheeks over the mandible, anterior to and below the ear; and the submandibular glands, which are beneath the mandible at the angle of the jaw. The parotid glands are not normally palpable.

In a patient who has anisocoria, the nurse would expect to observe: a. dilated pupils. b. excessive tearing. c. pupils of unequal size. d. an uneven curvature of the lens.

C Unequal pupil size is termed anisocoria. It exists normally in 5% of the population but may also be indicative of central nervous system disease

When examining the nares of a 45-year-old patient who has complaints of rhinorrhea, itching of the nose and eyes, and sneezing, the nurse notices the following: pale turbinates, swelling of the turbinates, and clear rhinorrhea. Which of these conditions is most likely the cause? a. Nasal polyps b. Acute sinusitis c. Allergic rhinitis d. Acute rhinitis

C With allergic rhinitis, rhinorrhea, itching of the nose and eyes, and sneezing are present. On physical examination, there is serous edema, and the turbinates usually appear pale with a smooth, glistening surface. See Table 16-1 for descriptions of the other conditions.

The nurse is preparing to perform a manual compression test on a patient. Which of these statements is true about this procedure? a. Rapid filling of the veins indicates incompetent veins. b. Competent valves in the veins will transmit a wave to the distal fingers. c. A palpable wave transmission occurs when the valves are incompetent. d. The test assesses whether the valves of varicosity are competent when the person is in the supine position.

C With the manual compression test, a palpable wave transmission occurs when the valves are incompetent. Competent veins will prevent a wave transmission and the nurse's distal (lower) fingers will feel no change. The test is performed while the patient is standing.

A patient has been diagnosed with venous stasis. Which of these findings would the nurse most likely observe? a. A unilateral cool foot b. Thin, shiny, atrophic skin c. Pallor of the toes and cyanosis of the nail beds d. A brownish discoloration to the skin of the lower leg

D A brown discoloration occurs with chronic venous stasis as a result of hemosiderin deposits (a by-product of red blood cell degradation). Pallor, cyanosis, atrophic skin, and unilateral coolness are all signs associated with arterial problems.

When auscultating over a patient's femoral arteries the nurse notices the presence of a bruit on the left side. The nurse knows that: a. bruits are often associated with venous disease. b. bruits occur in the presence of lymphadenopathy. c. hypermetabolic states will cause bruits in the femoral arteries. d. bruits occur with turbulent blood flow, indicating partial occlusion.

D A bruit occurs with turbulent blood flow and indicates partial occlusion of the artery. The other responses are not correct.

A patient comes into the emergency department after an accident at work. A machine blew dust into his eyes and he was not wearing safety glasses. The nurse examines his corneas by shining a light from the side across the cornea. What findings would suggest that he has suffered a corneal abrasion? a. Smooth and clear corneas b. Opacity of the lens behind the cornea c. Bleeding from the areas across the cornea d. A shattered look to the light rays reflecting off the cornea

D A corneal abrasion causes irregular ridges in reflected light, which produce a shattered look to light rays. There should be no opacities in the cornea. The other responses are not correct.

A mother brings her 4-month-old child to the clinic with concerns regarding a small pad in the middle of the upper lip that has been there since 1 month of age. The infant has no health problems. On physical examination, the nurse notices a 0.5-cm, fleshy, elevated area in the middle of the upper lip. There is no evidence of inflammation or drainage. What would the nurse tell this mother? a. "This is an area of irritation caused from teething and is nothing to worry about." b. "This is an abnormal finding and should be evaluated by another health care provider." c. "This is the result of chronic drooling and should resolve within the next month or two." d. "This is a sucking tubercle caused from the friction of breastfeeding or bottle-feeding and is normal."

D A normal finding in infants is the sucking tubercle, a small pad in the middle of the upper lip from the friction of breast- or bottle-feeding. This is not caused by irritation, teething, or excessive drooling, and evaluation by another health care provider is not warranted.

During the cardiac auscultation the nurse hears a sound occurring immediately after S2 at the second left intercostal space. To further assess this sound, what should the nurse do? a. Have the patient turn to the left side while the nurse listens with the bell. b. Ask the patient to hold his breath while the nurse listens again. c. No further assessment is needed because the nurse knows it is an S3. d. Watch the patient's respirations while listening for effect on the sound.

D A split S2 is a normal phenomenon that occurs toward the end of inspiration in some people. A split S2 is heard only in the pulmonic valve area, the second left interspace. When the split S2 is first heard, the nurse should not be tempted to ask the person to hold his or her breath so that the nurse can concentrate on the sounds. Breath holding will only equalize ejection times in the right and left sides of the heart and cause the split to go away. Instead, the nurse should concentrate on the split while watching the person's chest rise up and down with breathing.

A 19-year-old college student is brought to the emergency department with a severe headache he describes as "Like nothing I've ever had before." His temperature is 104° F, and he has a stiff neck. The nurse looks for other signs and symptoms of which problem? a. Head injury b. Cluster headache c. Migraine headache d. Meningeal inflammation

D Acute onset of neck stiffness and pain along with headache and fever occurs with meningeal inflammation. A severe headache in an adult or child who has never had it before is a red flag. Head injury and cluster or migraine headaches are not associated with a fever or stiff neck.

The nurse is performing an assessment. Which of these findings would cause the greatest concern? a. A painful vesicle inside the cheek for 2 days b. The presence of moist, nontender Stensen's ducts c. Stippled gingival margins that adhere snugly to the teeth d. An ulceration on the side of the tongue with rolled edges

D An ulceration on the side or base of the tongue or under the tongue raises the suspicion of cancer and must be investigated. Risk of early metastasis is present because of rich lymphatic drainage. The vesicle may be an aphthous ulcer, which is painful but not dangerous. The other responses are normal findings

A 70-year-old patient is scheduled for open-heart surgery. The surgeon plans to use the great saphenous vein for the coronary bypass grafts. The patient asks, "What happens to my circulation when the veins are removed?" The nurse should reply: a. "Venous insufficiency is a common problem after this type of surgery." b. "Oh, we have lots of veins-you won't even notice that it has been removed." c. "You will probably experience decreased circulation after the veins are removed." d. "Because the deeper veins in your leg are in good condition, this vein can be removed without harming your circulation."

D As long as the femoral and popliteal veins remain intact, the superficial veins can be excised without harming the circulation. The other

Which of these techniques best describes the test the nurse should use to assess the function of cranial nerve X? a. Observe the patient's ability to articulate specific words. b. Assess movement of the hard palate and uvula with the gag reflex. c. Have the patient stick out the tongue and observe for tremors or pulling to one side. d. Ask the patient to say "ahhh" and watch for movement of the soft palate and uvula.

D Ask the person to say "ahhh" and note that the soft palate and uvula rise in the midline. This tests one function of CN X, which is the vagus nerve. Cranial nerves IX and X are tested by eliciting the gag reflex. Cranial nerve XII is tested by asking the patient to stick out his or her tongue.

The findings from an assessment of a 70-year-old patient with swelling in his ankles include jugular venous pulsations 5 cm above the sternal angle when the head of his bed is elevated 45 degrees. The nurse knows that this finding indicates: a. decreased fluid volume. b. increased cardiac output. c. narrowing of jugular veins. d. elevated pressure related to heart failure.

D Because no cardiac valve exists to separate the superior vena cava from the right atrium, the jugular veins give information about activity on the right side of the heart. They reflect filling pressures and volume changes. Normal jugular venous pulsation is 2 cm or less above the sternal angle. Elevated pressure is more than 3 cm above the sternal angle at 45 degrees and occurs with heart failure.

When examining a 16-year-old male teenager, the nurse should: a. discuss health teaching with the parent because the teen is unlikely to be interested in promoting wellness. b. ask his parent to stay in the room during the history and physical examination to answer any questions and alleviate his anxiety. c. talk to him the same as one would talk would a younger child because a teen's level of understanding may not match his or her speech. d. provide feedback that his body is developing normally and discuss the wide variation among teenagers on the rate of growth and development.

D During the examination, the adolescent needs feedback that his or her body is healthy and developing normally. The adolescent has a keen awareness of body image and often compares himself or herself to peers. Apprise the adolescent of the wide variation among teenagers on the rate of growth and development.

A 52-year-old patient describes the presence of occasional "floaters" or "spots" moving in front of his eyes. The nurse should: a. examine the retina to determine the number of floaters. b. presume the patient has glaucoma and refer him for further testing. c. consider this an abnormal finding and refer him to an ophthalmologist. d. know that floaters are usually not significant and are caused by condensed vitreous fibers.

D Floaters are a common sensation with myopia or after middle age owing to condensed vitreous fibers. Usually they are not significant, but acute onset of floaters may occur with retinal de-tachment.

A 32-year-old woman is at the clinic for "little white bumps in my mouth." During the assessment, the nurse notes that she has a 0.5 cm white, nontender papule under her tongue and one on the mucosa of her right cheek. What would the nurse tell the patient? a. "These spots are seen with infections such as strep throat." b. "These could be indicative of a serious lesion, so I will refer you to a specialist." c. "This is called leukoplakia and can be caused by chronic irritation such as smoking." d. "These bumps are Fordyce's granules, which are sebaceous cysts and are not a serious condition."

D Fordyce's granules are small, isolated white or yellow papules on the mucosa of the cheek, tongue, and lips. These little sebaceous cysts are painless and are not significant. Chalky, white raised patches would indicate leukoplakia. In strep throat, the examiner would see tonsils that are bright red, swollen, and may have exudates or white spots.

A pregnant woman states that she is concerned about her gums because she has noticed they are swollen and have started bleeding. What would be an appropriate response by the nurse? a. "This is probably due to a vitamin C deficiency." b. "I'm not sure what causes it but let me know if it's not better in a few weeks." c. "You need to make an appointment with your dentist as soon as possible to have this checked." d. "This can be caused by the change in hormone balance in your system when you're pregnant."

D Gingivitis is when gum margins are red and swollen and bleed easily. The condition may occur in pregnancy and puberty because of a changing hormonal balance.

A patient complains that while studying for an examination he began to notice a severe headache in the frontotemporal area of his head that is throbbing and is somewhat relieved when he lies down. He tells the nurse that his mother also had these headaches. The nurse suspects that he may be suffering from: a. hypertension. b. cluster headaches. c. tension headaches. d. migraine headaches.

D Migraine headaches tend to be supraorbital, retro-orbital, or frontotemporal with a throbbing quality. They are of a severe quality and are relieved by lying down. Migraines are associated with family history of migraines.

During a cardiac assessment on a 38 year-old patient in the hospital for "chest pain," the nurse finds the following: jugular vein pulsations 4 cm above sternal angle when he is elevated at 45 degrees, blood pressure 98/60 mm Hg, heart rate 130 beats per minute, ankle edema, difficulty in breathing when supine, and an S3 on auscultation. Which of these conditions best explains the cause of these findings? a. Fluid overload b. Atrial septal defect c. Myocardial infarction d. Heart failure

D Heart failure causes decreased cardiac output when the heart fails as a pump and the circulation becomes backed up and congested. Signs and symptoms include dyspnea, orthopnea, paroxysmal nocturnal dyspnea, decreased blood pressure, dependent and pitting edema; anxiety; confusion; jugular vein distention; and fatigue. The S3 is associated with heart failure and is always abnormal after age 35. The S3 may be the earliest sign of heart failure.

A patient is unable to read even the largest letters on the Snellen chart. The nurse should take which action next? a. Refer the patient to an ophthalmologist or optometrist for further evaluation. b. Assess whether the patient can count the nurse's fingers when they are placed in front of his or her eyes. c. Ask the patient to put on his or her reading glasses and attempt to read the Snellen chart again. d. Shorten the distance between the patient and the chart until it is seen and record that distance.

D If the person is unable to see even the largest letters, then the nurse should shorten the distance to the chart until it is seen and should record that distance (e.g., "10/200"). If visual acuity is even lower, then the nurse should assess whether the person can count fingers when they are spread in front of the eyes or can distinguish light perception from a penlight. If vision is poorer than 20/30, then a referral to an ophthalmologist or optometrist is necessary, but first the nurse must assess the visual acuity.

During an assessment of the sclera of an African-American patient, the nurse would consider which of these an expected finding? a. Yellow fatty deposits over the cornea b. Pallor near the outer canthus of the lower lid c. Yellow color of the sclera that extends up to the iris d. The presence of small brown macules on the sclera

D In dark-skinned people, one normally may see small brown macules in the sclera.

When assessing a patient's pulse, the nurse notes that the amplitude is weaker during inspiration and stronger during expiration. When the nurse measures the blood pressure, the reading decreases 20 mm Hg during inspiration and increases with expiration. This patient is experiencing pulsus: a. alternans. b. bisferiens. c. bigeminus. d. paradoxus.

D In pulsus paradoxus, beats have a weaker amplitude with inspiration and a stronger amplitude with expiration. It is best determined during blood pressure measurement; reading decreases (>10 mm Hg) during inspiration and increases with expiration.

The nurse is assessing the lungs of an older adult. Which of these describes normal changes in the respiratory system of the older adult? a. Severe dyspnea is experienced on exertion resulting from changes in the lungs. b. Respiratory muscle strength increases to compensate for a decreased vital capacity. c. There is a decrease in small airway closure, leading to problems with atelectasis. d. The lungs are less elastic and distensible, which decreases their ability to collapse and recoil.

D In the aging adult the lungs are less elastic and distensible, which decreases their ability to collapse and recoil. There is a decreased vital capacity and a loss of intraalveolar septa, causing less surface area for gas exchange. The lung bases become less ventilated, and the older person is at risk for dyspnea with exertion beyond his or her usual workload

The nurse is obtaining a history on a 3-month-old infant. During the interview, the mother states, "I think she is getting her first tooth because she has started drooling a lot." The nurse's best response would be: a. "You're right, drooling is usually a sign of the first tooth." b. "It would be unusual for a 3 month old to be getting her first tooth." c. "This could be the sign of a problem with the salivary glands." d. "She is just starting to salivate and hasn't learned to swallow the saliva."

D In the infant, salivation starts at 3 months. The baby will drool for a few months before learning to swallow the saliva. This drooling does not herald the eruption of the first tooth, although many parents think it does.

When inspecting the anterior chest of an adult, the nurse should include which assessment? a. Diaphragmatic excursion b. Symmetric chest expansion c. The presence of breath sounds d. The shape and configuration of the chest wall

D Inspection of the anterior chest includes shape and configuration of the chest wall; assessment of the patient's level of consciousness, skin color and condition; quality of respirations; presence or absence of retraction and bulging of the intercostal spaces; and use of accessory muscles. Symmetric chest expansion is assessed by palpation. Diaphragmatic excursion is assessed by percussion of the posterior chest. Breath sounds are assessed by auscultation.

The nurse is reviewing causes of increased intraocular pressure. Which of these factors deter-mines intraocular pressure? a. Thickness or bulging of the lens b. Posterior chamber as it accommodates an increase in fluid c. Contraction of the ciliary body in response to the aqueous within the eye d. Amount of aqueous produced and resistance to its outflow at the angle of the anterior chamber

D Intraocular pressure is determined by a balance between the amount of aqueous produced and resistance to its outflow at the angle of the anterior chamber. The other responses are incorrect.

A mother brings her 2-month-old daughter in for an examination and says, "My daughter rolled over against the wall and now I have noticed that she has this spot that is soft on the top of her head. Is there something terribly wrong?" The nurse's best response would be: a. "Perhaps that could be a result of your dietary intake during pregnancy." b. "Your baby may have craniosynostosis, a disease of the sutures of the brain." c. "That 'soft spot' you are referring to may be an indication of cretinism or congenital hypothyroidism." d. "That 'soft spot' is normal, and actually allows for growth of the brain during the first year of your baby's life."

D Membrane-covered "soft spots" allow for growth of the brain during the first year. They gradually ossify; the triangular-shaped posterior fontanel is closed by 1 to 2 months, and the diamond-shaped anterior fontanel closes between 9 months and 2 years.

A 30-year-old woman with a history of mitral valve problems states that she has been "very tired." She has started waking up at night and feels like her "heart is pounding." During the assessment, the nurse palpates a thrill and lift at the fifth left intercostal space midclavicular line. In the same area the nurse also auscultates a blowing, swishing sound right after S1. These findings would be most consistent with: a. heart failure. b. aortic stenosis. c. pulmonary edema. d. mitral regurgitation.

D Mitral regurgitation subjective findings include fatigue, palpitation, and orthopnea. Objective findings are (1) a thrill in systole at apex, (2) lift at apex, (3) apical impulse displaced down and to the left, (4) S1 diminished, S2 accentuated, S3 at apex often present, and (5) murmur: pansystolic, often loud, blowing, best heard at apex, radiating well to the left axilla.

During ocular examinations, the nurse keeps in mind that movement of the extraocular muscles is: a. decreased in the elderly. b. impaired in a patient with cataracts. c. stimulated by cranial nerves I and II. d. stimulated by cranial nerves III, IV, and VI.

D Movement of the extraocular muscles is stimulated by three cranial nerves: III, IV, and VI.

The nurse is aware that the four areas in the body where lymph nodes are accessible are the: a. head, breasts, groin, and abdomen. b. arms, breasts, inguinal area, and legs. c. head and neck, arms, breasts, and axillae. d. head and neck, arms, inguinal area, and axillae.

D Nodes are located throughout the body, but they are accessible to examination only in four areas: head and neck, arms, inguinal region, and axillae

During inspection of the precordium of an adult patient, the nurse notices the chest moving in a forceful manner along the sternal border. This finding most likely suggests: a. a normal heart. b. a systolic murmur. c. enlargement of the left ventricle. d. enlargement of the right ventricle.

D Normally, the examiner may or may not see an apical impulse; when visible, it occupies the fourth or fifth intercostal space at or inside the midclavicular line. A heave or lift is a sustained forceful thrusting of the ventricle during systole. It occurs with ventricular hypertrophy as a result of increased workload. A right ventricular heave is seen at the sternal border; a left ventricular heave is seen at the apex.

The nurse is reviewing the development of the newborn infant. Regarding the sinuses, which statement is true in relation to a newborn infant? a. The sphenoid sinuses are full size at birth. b. The maxillary sinuses reach full size after puberty. c. The frontal sinuses are fairly well developed at birth. d. The maxillary and ethmoid sinuses are the only sinuses present at birth.

D Only the maxillary and ethmoid sinuses are present at birth. The sphenoid sinuses are minute at birth and develop after puberty. The frontal sinuses are absent at birth, are fairly well developed at age 7 to 8 years, and reach full size after puberty

An ophthalmic examination reveals papilledema. The nurse is aware that this finding indicates: a. retinal detachment. b. diabetic retinopathy. c. acute-angle glaucoma. d. increased intracranial pressure.

D Papilledema, or choked disk, is a serious sign of increased intracranial pressure, which is caused by a space-occupying mass such as a brain tumor or hematoma. This pressure causes venous stasis in the globe, showing redness, congestion, and elevation of the optic disc, blurred margins, hemorrhages, and absent venous pulsations. Papilledema is not associated with the conditions in the other responses.

The nurse hears bilateral louder, longer, and lower tones when percussing over the lungs of a 4-year-old child. What should the nurse do next? a. Palpate over the area for increased pain and tenderness. b. Ask the child to take shallow breaths and percuss over the area again. c. Refer the child immediately because of an increased amount of air in the lungs. d. Consider this a normal finding for a child this age and proceed with the examination.

D Percussion notes that are louder in amplitude, lower in pitch, of a booming quality, and longer in duration are normal over a child's lung.

When examining an infant, the nurse should examine which area first? a. Ear b. Nose c. Throat d. Abdomen

D Perform the least distressing steps first. Save the invasive steps of examination of the eye, ear, nose, and throat until last.

During a morning assessment, the nurse notices that the patient's sputum is frothy and pink. Which condition could this finding indicate? a. Croup b. Tuberculosis c. Viral infection d. Pulmonary edema

D Sputum alone is not diagnostic, but some conditions have characteristic sputum production. Pink, frothy sputum indicates pulmonary edema (or it may be a side effect of sympathomimetic medications). Croup is associated with a "barking" cough, not sputum production. Tuberculosis may produce rust-colored sputum. Viral infections may produce white or clear mucoid sputum.

The nurse is performing an eye-screening clinic at a daycare center. When examining a 2-year-old child, the nurse suspects that the child has "lazy eye" and should: a. examine the external structures of the eye. b. assess visual acuity with the Snellen eye chart. c. assess the child's visual fields with the confrontation test. d. test for strabismus by performing the corneal light reflex test.

D Testing for strabismus is done by performing the corneal light reflex test as well as the cover test. The Snellen eye chart and confrontation test are not used to test for strabismus

The nurse is assessing a patient's eyes for the accommodation response and would expect to see which normal finding? a. Dilation of the pupils b. A consensual light reflex c. Conjugate movement of the eyes d. Convergence of the axes of the eyes

D The accommodation reaction includes pupillary constriction and convergence of the axes of the eyes. The other responses are not correct.

During an assessment of a healthy adult, where would the nurse expect to palpate the apical impulse? a. Third left intercostal space at the midclavicular line b. Fourth left intercostal space at the sternal border c. Fourth left intercostal space at the anterior axillary line d. Fifth left intercostal space at the midclavicular line

D The apical impulse should occupy only one intercostal space, the fourth or fifth, and it should be at or medial to the midclavicular line

The nurse is preparing to assess the dorsalis pedis artery. Where is the correct location for palpation? a. Behind the knee b. Over the lateral malleolus c. In the groove behind the medial malleolus d. Lateral to the extensor tendon of the great toe

D The dorsalis pedis artery is located on the dorsum of the foot. The nurse should palpate just lateral to and parallel with the extensor tendon of the big toe. The popliteal artery is palpated behind the knee. The posterior tibial pulse is palpated in the groove between the malleolus and the Achilles tendon. There is no pulse palpated at the lateral malleolus.

The tissue that connects the tongue to the floor of the mouth is the: a. uvula. b. palate. c. papillae. d. frenulum.

D The frenulum is a midline fold of tissue that connects the tongue to the floor of the mouth. The uvula is the free projection hanging down from the middle of the soft palate. The palate is the arching roof of the mouth. Papillae are the rough, bumpy elevations on the tongue's dorsal surface.

An examiner is using an ophthalmoscope to examine a patient's eyes. The patient has astigmatism and is nearsighted. The use of which of these techniques would indicate that the examination is being performed correctly? a. Using the large full circle of light when assessing pupils that are not dilated b. Rotating the lens selector dial to the black numbers to compensate for astigmatism c. Using the grid on the lens aperture dial to visualize the external structures of the eye d. Rotating the lens selector dial to bring the object into focus

D The ophthalmoscope is used to examine the internal eye structures. It can compensate for nearsightedness or farsightedness, but it will not correct for astigmatism. The grid is used to assess size and location of lesions on the fundus. The large full spot of light is used to assess dilated pupils. Rotating the lens selector dial brings the object into focus.

A patient has a normal pupillary light reflex. The nurse recognizes that this reflex indicates that: a. the eyes converge to focus on the light. b. light is reflected at the same spot in both eyes. c. the eye focuses the image in the center of the pupil. d. constriction of both pupils occurs in response to bright light.

D The pupillary light reflex is the normal constriction of the pupils when bright light shines on the retina. The other responses are not correct.

The nurse is preparing to perform a physical assessment. The correct action by the nurse is reflected by which statement? The nurse: a. performs the examination from the left side of the bed. b. examines tender or painful areas first to help relieve the patient's anxiety. c. follows the same examination sequence regardless of the patient's age or condition. d. organizes the assessment so that the patient does not change positions too often.

D The steps of the assessment should be organized so that the patient does not change positions too often. The sequence of the steps of the assessment may differ depending on the age of the person and the examiner's preference. Tender or painful areas should be assessed last.

During an examination of the eye, the nurse would expect what normal finding when assessing the lacrimal apparatus? a. The presence of tears along the inner canthus b. A blocked nasolacrimal duct in a newborn infant c. A slight swelling over the upper lid and along the bony orbit if the individual has a cold d. The absence of drainage from the puncta when pressing against the inner orbital rim

D There should be no swelling, redness, or drainage from the puncta when it is pressed. Regurgitation of fluid from the puncta, when pressed, indicates duct blockage. The lacrimal glands are not functional at birth.

The mother of a 3-month-old infant states that her baby has not been gaining weight. With further questioning, the nurse finds that the infant falls asleep after nursing and wakes up after a short amount of time, hungry again. What other information would the nurse want to have? a. The infant's sleeping position b. Sibling history of eating disorders c. Amount of background noise when eating d. Presence of dyspnea or diaphoresis when sucking

D To screen for heart disease in an infant, focus on feeding. Note fatigue during feeding. An infant with heart failure takes fewer ounces each feeding, becomes dyspneic with sucking, may be diaphoretic and then falls into exhausted sleep and awakens after a short time hungry again.

The nurse is reviewing anatomy and physiology of the heart. Which statement best describes what is meant by atrial kick? a. The atria contract during systole and attempt to push against closed valves. b. The contraction of the atria at the beginning of diastole can be felt as a palpitation. c. This is the pressure exerted against the atria as the ventricles contract during systole. d. The atria contract toward the end of diastole and push the remaining blood into the ventricles.

D Toward the end of diastole, the atria contract and push the last amount of blood (about 25% of stroke volume) into the ventricles. This active filling phase is called presystole, or atrial systole, or sometimes the "atrial kick."

The nurse is preparing to assess the ankle-brachial index (ABI) of a patient. Which statement about the ABI is true? a. Normal ABI indices are from 0.50 to 1.0. b. The normal ankle pressure is slightly lower than the brachial pressure. c. The ABI is a reliable measurement of peripheral vascular disease in diabetic individuals. d. An ABI of 0.90 to 0.70 indicates the presence of peripheral vascular disease and mild claudication.

D Use of the Doppler stethoscope is a noninvasive way to determine the extent of peripheral vascular disease. The normal ankle pressure is slightly greater than or equal to the brachial pressure. An ABI of 0.90 to 0.70 indicates the presence of peripheral vascular disease and mild claudication. The ABI is less reliable in patients with diabetes mellitus because of claudication, which makes the arteries noncompressible and may give a falsely high ankle pressure.

A 72-year-old patient has a history of hypertension and chronic lung disease. An important question for the nurse to include in the history would be: a. "Do you use a fluoride supplement?" b. "Have you had tonsillitis in the last year?" c. "At what age did you get your first tooth?" d. "Have you noticed any dryness in your mouth?"

D Xerostomia (dry mouth) is a side effect of many drugs used by older people, including antidepressants, anticholinergics, antispasmodics, antihypertensives, antipsychotics, bronchodilators


संबंधित स्टडी सेट्स

psych final part 1 chapter 1,2 & 3

View Set